Nothing Special   »   [go: up one dir, main page]

SFG 2024 - LEVEL 1 - Test 41

Download as pdf or txt
Download as pdf or txt
You are on page 1of 43

SFG 2024 | LEVEL 1 |Test 41 |

DO NOT OPEN THIS TEST BOOKLET UNTIL YOU ARE TOLD TO DO SO


T.B.C.: FIAS-SFG24-L1T41 Test Booklet Series
Serial No.

A
TEST BOOKLET
GENERAL STUDIES

Time Allowed: ONE HOUR Maximum Marks: 100


INSTRUCTIONS
1. IMMEDIATELY AFTER THE COMMENCEMENT OF THE EXAMINATION, YOU SHOULD CHECK
THAT THIS TEST BOOKLET DOES NOT HAVE ANY UNPRINTED OR TORN OR MISSING PAGES
OR ITEMS ETC. IF SO, GET IT REPLACED BY A COMPLETE TEST BOOK.
2. Please note that it is the candidate’s responsibility to encode and fill in the Roll Number and
Test Booklet Series A, B, C or D carefully without any omission or discrepancy at the
appropriate places in the OMR Answer Sheet. Any omission/discrepancy will render the Answer
Sheet liable for rejection.
3. You have to enter your Name, Email Id and
Name:
Mobile No. on the test booklet in the Box Email Id:
provided alongside. DO NOT write anything
Mobile No:
else on the Test Booklet.
4. This Test Booklet contains 50 items (questions). Each item is printed in English. Each item
comprises four responses (answers). You will select the response which you want to mark on
the Answer Sheet. In case you feel that there is more than one correct response, mark the
response which you consider the best. In any case, choose ONLY ONE response for each item.
5. You have to mark all your responses ONLY on the separate Answer Sheet provided.
See directions in the Answer Sheet.
6. All items carry equal marks.
7. Before you proceed to mark in the Answer Sheet the response to various items in the Test
Booklet, you have to fill in some particulars in the Answer Sheet as per instructions sent to you
with your Admission Certificate.
8. After you have completed filling in all your responses on the Answer Sheet and the examination
has concluded, you should hand over to the Invigilator only the Answer Sheet. You are
permitted to take away with you the Test Booklet.
9. Sheets for rough work are appended in the Test Booklet at the end.
10. Penalty for wrong answers:
THERE WILL BE PENALTY FOR WRONG ANSWERS MARKED BY A CANDIDATE IN THE
OBJECTIVE TYPE QUESTION PAPERS.
(i) There are four alternatives for the answer to every question. For each question for which
a wrong answer has been given by the candidate, one-third of the marks assigned to that
question will be deducted as penalty.
(ii) If a candidate gives more than one answer, it will be treated as wrong answer even if one
of the given answers happens to be correct and there will be same penalty as above to
that question.
(iii) If a question is left bank i.e., no answer is given by the candidate, there will be no
O

penalty for that question.


C
S.
TE
O
FN

Forum Learning Centre: Delhi - 2nd Floor, IAPL House, 19 Pusa Road, Karol Bagh, New Delhi - 110005 | Patna - 2nd floor, AG Palace, E Boring Canal
Road, Patna, Bihar 800001 | Hyderabad - 1st & 2nd Floor, SM Plaza, RTC X Rd, Indira Park Road, Jawahar Nagar, Hyderabad, Telangana 500020
D

9311740400, 9311740900 | https://academy.forumias.com | admissions@forumias.academy | helpdesk@forumias.academy


.P
W

[1]
W
W
SFG 2024 | LEVEL 1 |Test 41 |

Q.1) Which of the following are detritivores? Q.4) Consider the following:
1. Earthworms 1. Protozoa.
2. Jellyfish 2. Virus.
3. Millipedes 3. Bacteria.
4. Seahorses 4. Lightning and thunder.
5. Woodlice How many of the above play a crucial role in
Select the correct answer using the code given converting atmospheric nitrogen into a
below: compound effectively utilized by plants?

a) 1, 2 and 4 only a) Only one

b) 2, 3, 4 and 5 only b) Only two

c) 1, 3 and 5 only c) Only three

d) 1, 2, 3, 4 and 5 d) All four

Q.2) Which one of the following correctly Q.5) Consider the following statements:
describes the meaning of Net Primary 1. Tundra region is found in the higher
Productivity of an ecosystem? latitudes near the polar regions only.
a) Total amount of organic matter produced 2. Cotton grass and lichens are the vegetation
by autotrophs, by the process of found in the arctic Tundra region.
Photosynthesis. 3. Mammals in the tundra region have small
b) Total energy captured by the primary tails and small ears.
producers through photosynthesis minus How many of the statements given above are
the losses due to respiration. correct?
c) Total amount of energy transferred from a) Only one
one trophic level to the next trophic level in b) Only two
a food chain. c) All three
d) Total amount of organic matter produced d) None
at all trophic levels in an ecosystem.

Q.6) Consider the following statements


Q.3) It is an ecological concept that describes regarding „Kelp forests‟:
how there is a greater diversity of life in the
1. They are underwater forests dominated by
region where the edges of two adjacent
brown algae.
ecosystems overlap, such as land/water, or
2. They occur only in tropical warm waters as
forest/grassland. At the interface of two
they cannot thrive in cold waters.
overlapping ecosystems, we can find species
from both of these ecosystems, as well as 3. They play a crucial role in reducing the rate
unique species that aren‟t found in either of Coastal erosion.
ecosystem but are specially adapted to the 4. They are a valuable source of alginates that
conditions of the transition zone. are used in various food industries.
Which one of the following ecological How many of the statements given above are
concepts is described in the above given correct?
paragraph? a) Only one
a) Ecological resilience b) Only two
b) Population Threshold c) Only three
c) Edge Effect d) All four
d) Keystone species
O
C
S.
TE
O
FN

Forum Learning Centre: Delhi - 2nd Floor, IAPL House, 19 Pusa Road, Karol Bagh, New Delhi - 110005 | Patna - 2nd floor, AG Palace, E Boring Canal
Road, Patna, Bihar 800001 | Hyderabad - 1st & 2nd Floor, SM Plaza, RTC X Rd, Indira Park Road, Jawahar Nagar, Hyderabad, Telangana 500020
D

9311740400, 9311740900 | https://academy.forumias.com | admissions@forumias.academy | helpdesk@forumias.academy


.P
W

[2]
W
W
SFG 2024 | LEVEL 1 |Test 41 |

Q.7) With reference to biotic interactions of Q.10) Which of the following statements
living organisms, consider the following pairs: explains the meaning of the „Carrying Capacity‟
Types of Examples of an ecosystem?
interactions a) It refers to the amount of resources
1. Mutualism Honeybees and flowers consumed by each individual within the
2. Commensalism A large tree shades a small ecosystem.
plant b) It is the maximum population size that an
3. Amensalism Cow dung and dung ecosystem can support without degrading
beetles. itself.
4. Parasitism Virus inside human body c) It refers to the total size of renewable
How many of the pairs given above are resources available in the ecosystem.
correctly matched? d) It is the sum of all the physical and
a) Only one chemical factors that a species needs to
b) Only two survive in an ecosystem.
c) Only three
d) All four Q.11) Which of the following have species that
can establish symbiotic relationship with other
Q.8) Consider the following statements organisms?
regarding an environmental pollutant: 1. Cnidarians
1. It is water-soluble. 2. Fungi
2. It is biologically active. 3. Protozoa
3. It is immobile. Select the correct answer using the code given
How many of the above characteristics are below:
essential for an environmental pollutant to a) 1 and 2 only
undergo biomagnification? b) 2 and 3 only
a) Only one c) 1 and 3 only
b) Only two d) 1, 2 and 3
c) All three
d) None Q.12) In the context of Ecology, which one of
the following best describes the meaning of
Q.9) With reference to the characteristics of the term 'Population'?
humus, consider the following statements: Population is a group of-
Statement-I: Humus undergoes a) individuals of the same species living in the
decomposition at an extremely slow rate. same geographical area at a given point of
Statement-II: Humus is highly resistant to time.
microbial action. b) different species living in the same
Which one of the following is correct in geographical area at a given point of time.
respect of the above statements? c) organisms that can interbreed and
a) Both Statement-I and Statement-II are reproduce with one another at a given
correct and Statement-II is not the correct point of time.
explanation for Statement-I. d) all living organisms that inhabit the earth at
b) Both Statement-I and Statement-II are a given point of time.
correct and Statement-II is the correct
explanation for Statement-I.
c) Statement-I is incorrect but Statement-II is
correct.
d) Statement-I is correct but Statement-II is
incorrect.
O
C
S.
TE
O
FN

Forum Learning Centre: Delhi - 2nd Floor, IAPL House, 19 Pusa Road, Karol Bagh, New Delhi - 110005 | Patna - 2nd floor, AG Palace, E Boring Canal
Road, Patna, Bihar 800001 | Hyderabad - 1st & 2nd Floor, SM Plaza, RTC X Rd, Indira Park Road, Jawahar Nagar, Hyderabad, Telangana 500020
D

9311740400, 9311740900 | https://academy.forumias.com | admissions@forumias.academy | helpdesk@forumias.academy


.P
W

[3]
W
W
SFG 2024 | LEVEL 1 |Test 41 |

Q.13) Consider the following statements Q.16) With reference to the various Biomes and
regarding „PARIVESH‟ portal of the their characteristics, consider the following
Government of India: pairs:
1. It has been developed by the Ministry of
Biome Characteristics
Environment, Forest and Climate Change
(MoEF&CC).
1. Taiga Highest Productivity among
2. It provides for online submission and
forest ecosystems.
monitoring of the proposals seeking
Environment and Wildlife clearance.
2. Tropical Poor nutrient composition of
Which of the statements given above is/are Rainforests the soil.
correct?
a) 1 only 3. Savannah Distinct Wet and Dry Season.
b) 2 only Grassland
c) Both 1 and 2
How many of the above pairs are correctly
d) Neither 1 nor 2 matched?
a) Only one
Q.14) Consider the following ecosystems: b) Only two
1. Lake c) All three
2. Wetland d) None
3. Savannah Grassland
4. Oceans Q.17) With reference to the Environmental
5. Desert Impact Assessment (EIA), consider the
Which of the options given below correctly following statements:
arranges ecosystem in decreasing order of 1. The objective of EIA is to foresee the
their Net Primary Productivity (NPP)? potential environmental problems that
a) 2-1-4-3-5 would arise out of a proposed development
b) 2-3-4-1-5 and address them in the project's planning
c) 2-1-3-5-4 and design stage.

d) 2-3-1-4-5 2. The legal basis for the EIA is provided


under the Environmental (Protection) Act,
1986.
Q.15) Which of the following statements
Which of the statements given above is/are
correctly describes the term „Ecological
correct?
footprint‟?
a) 1 only
a) The amount of carbon-dioxide produced
because of any activity by a human or b) 2 only
machine. c) Both 1 and 2
b) The amount of oxygen required to d) Neither 1 nor 2
replenish the environment in a particular
area.
c) The total greenhouse gas (GHG) emissions
caused by an individual, organization or
place expressed as carbon dioxide
equivalent.
d) The area needed to provide the renewable
resources that a population consumes and
O
C

to absorb its waste.


S.
TE
O
FN

Forum Learning Centre: Delhi - 2nd Floor, IAPL House, 19 Pusa Road, Karol Bagh, New Delhi - 110005 | Patna - 2nd floor, AG Palace, E Boring Canal
Road, Patna, Bihar 800001 | Hyderabad - 1st & 2nd Floor, SM Plaza, RTC X Rd, Indira Park Road, Jawahar Nagar, Hyderabad, Telangana 500020
D

9311740400, 9311740900 | https://academy.forumias.com | admissions@forumias.academy | helpdesk@forumias.academy


.P
W

[4]
W
W
SFG 2024 | LEVEL 1 |Test 41 |

Q.18) With reference to the Mangroves a) The water cycle in wetlands involves
Ecosystem, consider the following statements: surface runoff, subsoil percolation and
Statement-I: Mangrove forests can survive in evaporation.
anaerobic conditions. b) Algae form the nutrient base upon which
Statement-II: Mangroves have a specialized fish, crustaceans, molluscs, birds, reptiles
root structure called Pneumatophores. and mammals thrive.
Which one of the following is correct in c) Wetlands play a vital role in maintaining
respect of the above statements? sedimentation balance and soil
a) Both Statement-I and Statement-II are stabilization.
correct and Statement-II is the correct d) Aquatic plants absorb heavy metals and
explanation for Statement-I excess nutrients.
b) Both Statement-I and Statement-II are
correct and Statement-II is not the correct Q.22) With reference to the various examples
explanation for Statement-I of food chain, consider the following
c) Statement-1 is correct but Statement-II is statements:
incorrect 1. At the lowest tropic level, the grazing Food
d) Statement-l is incorrect but Statement-II is Chain starts with plants whereas the
correct Detritus Food Chain starts with dead
organic matter.
Q.19) Consider the following kinds of 2. Instead of Grazing Food Chain, the Detritus
organisms: food chain acts as a major conduit of
1. Bacteria energy flow in an aquatic ecosystem.
2. Fungi Which of the statements given above is/are
3. Flowering plants correct?
Some species of which of the above kinds of a) 1 only
organisms are employed as biopesticides? b) 2 only
a) 1 only c) Both 1 and 2
b) 2 and 3 only d) Neither 1 nor 2
c) 1 and 3 only
d) 1, 2 and 3 Q.23) Consider the following statements:
1. The pioneer species are those species
Q.20) With reference to Sulphur cycle, which initiate the development of an
consider the following statements: ecological community in an area with
1. It is a sedimentary cycle as it does not currently no life form‟s existence.
involve gaseous components. 2. Pioneer species can tolerate harsh
2. Sulphur is taken by plants in the form of environmental conditions.
sulphates. 3. In pioneer community, the plants and
Which of the statements given above is/are animals are in balance with each other and
correct? their environment.
a) 1 only How many of the statements given above are
b) 2 only incorrect?
c) Both 1 and 2 a) Only one
d) Neither 1 nor 2 b) Only two
c) All three
Q.21) “If rainforests and tropical forests are the d) None
lungs of the Earth, then surely wetlands
function as its kidneys.” Which one of the
following functions of wetlands best reflects
O

the above statement?


C
S.
TE
O
FN

Forum Learning Centre: Delhi - 2nd Floor, IAPL House, 19 Pusa Road, Karol Bagh, New Delhi - 110005 | Patna - 2nd floor, AG Palace, E Boring Canal
Road, Patna, Bihar 800001 | Hyderabad - 1st & 2nd Floor, SM Plaza, RTC X Rd, Indira Park Road, Jawahar Nagar, Hyderabad, Telangana 500020
D

9311740400, 9311740900 | https://academy.forumias.com | admissions@forumias.academy | helpdesk@forumias.academy


.P
W

[5]
W
W
SFG 2024 | LEVEL 1 |Test 41 |

Q.24) With reference to Sundarbans Forest Q.27) Consider the following statements:
located in West Bengal, consider the following 1. Eurythermal organisms cannot tolerate any
statements: change in the temperatures.
1. It is bound to its west by the river 2. Hydrophytes are the plants that are able to
Muriganga. survive in anaerobic environments.
2. It is the only mangrove forest in the world 3. Epiphytes are the plants that use the
inhabited by Tigers. supporting plants for shelter and not for
3. Sundarbans National Park is included in the water or food.
UNESCO‟s World Heritage Site list. How many of the statements given above are
How many of the above statements are correct?
correct? a) Only One
a) Only one b) Only two
b) Only two c) All three
c) All three d) None
d) None
Q.28) In the context of the Wetlands
Q.25) An ecosystem is the basic unit for the (Conservation and Management) Rules of 2017,
scientific study of nature. The ecosystem is consider the following:
made up of two inseparable components 1. River Channels
which are biotope and biocenosis. In this 2. Paddy fields
context, consider the following statements: 3. Man-made water bodies specifically for
1. Biocenosis represents the non-living drinking purposes
components of an ecosystem with specific 4. Structures specifically constructed for
physical characteristics. aquaculture purposes
2. Biotope represents the living components 5. Marsh ecosystem
of an ecosystem where living organisms are How many of the above are not included in the
in constant interaction. definition of „wetland‟ as per the provisions of
Which of the statements given above is/are the Wetlands (Conservation and Management)
correct? Rules of 2017?
a) 1 only a) Only two
b) 2 only b) Only three
c) Both 1 and 2 c) Only four
d) Neither 1 nor 2 d) All five

Q.26) Consider the following statements Q.29) What would happen if phytoplankton of
regarding the „Forest Advisory Committee‟: an ocean is completely destroyed for some
1. It is the final authority to approve the reason?
proposals for the diversion of forest land 1. The ocean as a carbon sink would be
for non-forest uses in India. adversely affected.
2. It is a statutory body incorporated under 2. The food chains in the ocean would be
the Forest Rights Act of 2006. adversely affected.
3. The Minister of Environment, Forest and 3. The density of ocean water would
Climate Change is the ex-officio drastically decrease.
Chairperson of this Committee. Select the correct answer using the codes
How many of the above statements are given below:
correct? a) 1 and 2 only
a) Only one b) 2 only
b) Only two c) 3 only
O

c) All three d) 1, 2 and 3


C
S.

d) None
TE
O
FN

Forum Learning Centre: Delhi - 2nd Floor, IAPL House, 19 Pusa Road, Karol Bagh, New Delhi - 110005 | Patna - 2nd floor, AG Palace, E Boring Canal
Road, Patna, Bihar 800001 | Hyderabad - 1st & 2nd Floor, SM Plaza, RTC X Rd, Indira Park Road, Jawahar Nagar, Hyderabad, Telangana 500020
D

9311740400, 9311740900 | https://academy.forumias.com | admissions@forumias.academy | helpdesk@forumias.academy


.P
W

[6]
W
W
SFG 2024 | LEVEL 1 |Test 41 |

Q.30) Consider the following statements about Q.33) Consider the following statements with
Euphotic Zone/ Photic zone: reference to Harmful Algal Blooms (HAB):
1. It is the upper layer of the aquatic 1. It can be caused by the upwelling of
ecosystems, up to which light penetrates seawater.
and within which photosynthetic activity is 2. It can lead to discoloration of water.
confined. 3. It can lead to an increase of dinoflagellates
2. It is also known as the hadopelagic zone. and diatoms in water bodies.
3. Common species that can be primarily How many of the statements given above are
found in euphotic zones are scallops, sea correct?
urchins, shrimps and krills. a) Only one
How many of the statements given above are b) Only two
correct? c) All three
a) Only one d) None
b) Only two
c) All three Q.34) Which of the following statements best
d) None describes the term „Standing Crop‟ in the
context of Ecology?
Q.31) Which of the following statements best a) It is the biomass of all primary producers in
describes “carbon fertilization”? a unit area at a specific moment in time.
a) Increased plant growth due to increased b) It is the total biomass of crops sown in a
concentration of carbon dioxide in the hectare at a specific moment in time.
atmosphere. c) It is the biomass of all trees in a forest
b) Increased temperature of Earth due to ecosystem at a specific moment in time.
increased concentration of carbon dioxide d) It is the biomass of all living organisms in a
in the atmosphere. unit area at a specific moment in time.
c) Increased acidity of oceans as a result of
increased concentration of carbon dioxide Q.35) Consider the following statements
in the atmosphere. regarding the „shola‟ forests:
d) Adaptation of all living beings on Earth to 1. These are categorized as montane wet
the climate change brought about by the temperate forests.
increased concentration of carbon dioxide 2. These forests can be found in the Nilgiris
in the atmosphere. ranges of Tamil Nadu and Kerala.
Which of the statements given above is/are
Q.32) In every summer we see a rise in the correct?
events of Forest Fires. In this context consider a) 1 only
the following: b) 2 only
1. Recycling of Nutrients. c) Both 1 and 2
2. Proliferation of Invasive species. d) Neither 1 nor 2
3. Reduction in the competition among the
existing tree species.
4. Facilitating seed germination.
5. Creating diverse habitats for plants and
animals.
How many of the above given are the
consequences of forest fire?
a) Only two
b) Only three
c) Only four
O

d) All five
C
S.
TE
O
FN

Forum Learning Centre: Delhi - 2nd Floor, IAPL House, 19 Pusa Road, Karol Bagh, New Delhi - 110005 | Patna - 2nd floor, AG Palace, E Boring Canal
Road, Patna, Bihar 800001 | Hyderabad - 1st & 2nd Floor, SM Plaza, RTC X Rd, Indira Park Road, Jawahar Nagar, Hyderabad, Telangana 500020
D

9311740400, 9311740900 | https://academy.forumias.com | admissions@forumias.academy | helpdesk@forumias.academy


.P
W

[7]
W
W
SFG 2024 | LEVEL 1 |Test 41 |

Q.36) Read the following paragraph carefully c) All four


and answer the question that follows: d) None
“When Arizona‟s Grand Canyon was formed, it
acted as a huge physical barrier and resulted in Q.39) Which one of the following terms
the separation of the species of squirrels into describes NOT only the physical space
two distinct groups of almost equal sizes. The occupied by an organism, but also its
barrier made it impossible for the species to functional role in the community of
breed with one another. Now, each species, organisms?
equally dominant, develops differently based
a) Ecotone
on the demand of their unique habitat.”
b) Ecological niche
Which of the following type of speciation most
c) Habitat
appropriately describes the above paragraph?
d) Home range
a) Allopatric Speciation
b) Peripatric Speciation
Q.40) Consider the following statements
c) Parapatric Speciation
regarding aquatic ecosystems:
d) Artificial Speciation
1. The Lentic aquatic system includes
freshwater streams, springs and rivers.
Q.37) Consider the following statements about
2. Estuaries are more productive as compared
ecological pyramids:
to the adjacent sea or river.
1. It does not include the possibility of a
Which of the statements given above is/are
species to be present at two or more
correct?
trophic levels.
a) 1 only
2. It does not accommodate a food web.
b) 2 only
3. Saprophytes are often not included in
c) Both 1 and 2
ecological pyramids.
d) Neither 1 nor 2
How many of the above are limitations of
ecological pyramids?
a) Only one Q.41) Due to some reasons, if there is a huge
fall in the population of species of butterflies,
b) Only two
what could be its likely consequence/
c) All three
consequences?
d) None
1. Pollination of some plants could be
adversely affected.
Q.38) In the context of Banni Grasslands, 2. There could be a drastic increase in the
consider the following statements: fungal infections of some cultivated plants.
1. It is found in Kachchh district in the state 3. It could lead to a fall in the population of
of Gujarat. some species of wasps, spiders and birds.
2. It represents the combination of wetland Select the correct using the code given below:
and grassland ecosystem at the same place.
a) 1 only
3. Bhutia tribe dominate the Banni grassland
b) 2 and 3 only
area.
c) 1 and 3 only
4. It has been approved to host a Cheetah
d) 1, 2 and 3
conservation breeding center.
How many of the statements given above are
correct?
a) Only two
O
C

b) Only three
S.
TE
O
FN

Forum Learning Centre: Delhi - 2nd Floor, IAPL House, 19 Pusa Road, Karol Bagh, New Delhi - 110005 | Patna - 2nd floor, AG Palace, E Boring Canal
Road, Patna, Bihar 800001 | Hyderabad - 1st & 2nd Floor, SM Plaza, RTC X Rd, Indira Park Road, Jawahar Nagar, Hyderabad, Telangana 500020
D

9311740400, 9311740900 | https://academy.forumias.com | admissions@forumias.academy | helpdesk@forumias.academy


.P
W

[8]
W
W
SFG 2024 | LEVEL 1 |Test 41 |

Q.42) With reference to the Phosphorous a) Only one


cycle, consider the following statements: b) Only two
1. It is a sedimentary biogeochemical cycle. c) Only three
2. Unlike the Carbon cycle, there is no d) All four
respiratory release of Phosphorous into the
atmosphere. Q.46) With reference to food chain, which of
3. Phosphorus is added to the environment by the following statements correctly explains the
weathering rocks. Lindemann law?
How many of the above statements are a) In a food chain only 10 per cent of the
correct? energy is transferred from the Sun to the
a) Only one lowest trophic level.
b) Only two b) In the detritus food chain, only 10 per cent
c) All three of the energy is transferred to each trophic
d) None level from the lower trophic level.
c) In grazing food chain, only 10 per cent of
Q.43) In the context of India State of Forest the energy is transferred to each trophic
Report (ISFR), 2021, which of the following level from the lower trophic level.
statements is incorrect? d) In a food chain, minimum 10 per cent of the
a) Arunachal Pradesh has recorded the energy is transferred back to the
maximum amount of forest carbon stock. environment.
b) Assam has recorded the maximum area
under bamboo cultivation. Q.47) Consider the following sites in India:
c) West Bengal has the maximum area 1. Gulf of Mannar
covered under the mangrove vegetation. 2. Sunderbans
d) Mizoram has the maximum forest area 3. Gulf of Kutch
categorized as an „extreme fire-prone‟ At how many of the above-mentioned sites,
zone. both mangroves and coral reefs can be
simultaneously found?
Q.44) Consider the following functions: a) Only one
1. Purification of water. b) Only two
2. Providing habitat for marine organisms. c) All three
3. To provide suitable location for d) None
development of seaports.
4. To act as natural buffers for the coastal Q.48) Consider the following statements:
areas against storms. 1. Eco-sensitivity is characterized by low
How many of the above are the functions of levels of resilience that cannot be restored
the Estuarine Ecosystem? to its original form easily.
a) Only one 2. An organism's ability to adjust its
b) Only two phenotype to new environmental
c) Only three conditions over its lifetime is called
d) All four Acclimatization.
Which of the above statements given above
Q.45) Consider the following factors: is/are correct?
1. Excessive use of fertilizers and pesticides a) 1 only
2. Excessive Mining of natural resources b) 2 only
3. Rampant Urbanization c) Both 1 and 2
4. Over-drafting of Groundwater d) Neither 1 nor 2
How many of the above given factors can
O

promote the process of desertification of land


C
S.

in the long run?


TE
O
FN

Forum Learning Centre: Delhi - 2nd Floor, IAPL House, 19 Pusa Road, Karol Bagh, New Delhi - 110005 | Patna - 2nd floor, AG Palace, E Boring Canal
Road, Patna, Bihar 800001 | Hyderabad - 1st & 2nd Floor, SM Plaza, RTC X Rd, Indira Park Road, Jawahar Nagar, Hyderabad, Telangana 500020
D

9311740400, 9311740900 | https://academy.forumias.com | admissions@forumias.academy | helpdesk@forumias.academy


.P
W

[9]
W
W
SFG 2024 | LEVEL 1 |Test 41 |

Q.49) Consider the following statements with


reference to Eutrophic lakes and Oligotrophic
lakes:
1. Unlike Eutrophic lakes, Oligotrophic lakes
are rich in nutrients.
2. Biological Oxygen Demand in Oligotrophic
Lake is higher as compared to Eutrophic
Lake.
3. Eutrophic lakes typically have reduced
water quality whereas oligotrophic lakes
have clearer water.
How many of the statements given above are
correct?
a) Only one
b) Only two
c) All three
d) None

Q.50) Within biological communities, some


species are important in determining the
ability of a large number of other species to
persist in the community. Such species are
called:
a) Keystone species
b) Allopatric species
c) Sympatric species
d) Threatened species

O
C
S.
TE
O
FN

Forum Learning Centre: Delhi - 2nd Floor, IAPL House, 19 Pusa Road, Karol Bagh, New Delhi - 110005 | Patna - 2nd floor, AG Palace, E Boring Canal
Road, Patna, Bihar 800001 | Hyderabad - 1st & 2nd Floor, SM Plaza, RTC X Rd, Indira Park Road, Jawahar Nagar, Hyderabad, Telangana 500020
D

9311740400, 9311740900 | https://academy.forumias.com | admissions@forumias.academy | helpdesk@forumias.academy


.P
W

[10]
W
W
SFG 2024 | LEVEL 1 | Test #41 – Solutions |

Q.1) Which of the following are detritivores?


1. Earthworms
2. Jellyfish
3. Millipedes
4. Seahorses
5. Woodlice
Select the correct answer using the code given below:
a) 1, 2 and 4 only
b) 2, 3, 4 and 5 only
c) 1, 3 and 5 only
d) 1, 2, 3, 4 and 5

Ans) c
Exp) Option c is the correct answer.
Detritivores are heterotrophs that obtain their nutrition by feeding on detritus i.e. dead or decaying
plants or animals.
Options 1, 3 and 5 are correct.
Earthworms, Millipedes and Woodlice are detritivores. Detritivores include microorganisms such as
bacteria and fungi; invertebrate insects such as mites, beetles, butterflies and flies; molluscs such as
slugs and snails; or soil-dwelling earthworms, millipedes and woodlice.
Options 2 and 4 are incorrect: Jellyfish is carnivorous and Sea horse is also primarily a carnivorous
animal.
Source: UPSC CSE PRE. 2021
Subject:) Environment
Subtopic:) Ecology and Ecosystems

Q.2) Which one of the following correctly describes the meaning of Net Primary Productivity of an
ecosystem?
a) Total amount of organic matter produced by autotrophs, by the process of Photosynthesis.
b) Total energy captured by the primary producers through photosynthesis minus the losses due to
respiration.
c) Total amount of energy transferred from one trophic level to the next trophic level in a food chain.
d) Total amount of organic matter produced at all trophic levels in an ecosystem.

Ans) b
Exp) Option b is the correct answer.
Productivity, in the context of ecosystems, refers to the rate at which organic matter is produced
through biological processes. This organic matter is mainly generated by autotrophic organisms, such
as plants, through photosynthesis. This process serves as the foundation of the ecosystem's energy
flow and sustains various trophic levels.
Option a is incorrect: Gross Primary Productivity (GPP) represents the total amount of organic
matter produced by autotrophs in an ecosystem through photosynthesis. GPP accounts for all the
energy assimilated by plants.
Option b is correct: Net Primary Productivity (NPP) is the total energy captured by primary
producers through photosynthesis minus losses due to respiration. NPP is the net gain in organic
matter that contributes to the growth and reproduction of plants and is available as food for
herbivores i.e. Organism in the next level of the food chain.
Option c is incorrect: Net Primary Productivity (NPP) pertains to the net energy accumulated by
O

autotrophs for their own sustenance, rather than representing the net energy transfer from one
C
S.

trophic level to the next trophic level in a food chain.


TE
O
FN

Forum Learning Centre: Delhi - 2nd Floor, IAPL House, 19 Pusa Road, Karol Bagh, New Delhi - 110005 | Patna - 2nd floor, AG Palace, E Boring Canal
Road, Patna, Bihar 800001 | Hyderabad - 1st & 2nd Floor, SM Plaza, RTC X Rd, Indira Park Road, Jawahar Nagar, Hyderabad, Telangana 500020
D

9311740400, 9311740900 | https://academy.forumias.com | admissions@forumias.academy | helpdesk@forumias.academy


.P
W

[1]
W
W
SFG 2024 | LEVEL 1 | Test #41 – Solutions |

Option d is incorrect: Net Primary Productivity (NPP) only considers the organic matter created by
primary producers (autotrophs), not all organisms across various trophic levels.
Source: https://ncert.nic.in/textbook/pdf/lebo114.pdf
Subject:) Environment
Subtopic:) Ecology and Ecosystems

Q.3) It is an ecological concept that describes how there is a greater diversity of life in the region
where the edges of two adjacent ecosystems overlap, such as land/water, or forest/grassland. At the
interface of two overlapping ecosystems, we can find species from both of these ecosystems, as well
as unique species that aren‟t found in either ecosystem but are specially adapted to the conditions of
the transition zone.
Which one of the following ecological concepts is described in the above given paragraph?
a) Ecological resilience
b) Population Threshold
c) Edge Effect
d) Keystone species

Ans) c
Exp) Option c is the correct answer.
Edge effect refers to the changes in population or community structures that occur at the boundary
of two habitats (ecotone). Sometimes the number of species and the population density of some of
the species in the ecotone is much greater than either community. This is called edge effect.
Edge Effect describes how there is a greater diversity of life in the region where the edges two
adjacent ecosystems overlap, such as land/water, or forest/grassland. At the interface of two
overlapping ecosystems, we can find species from both of these ecosystems, as well as unique species
that aren‟t found in either ecosystem but are specially adapted to the conditions of the transition
zone.
Option a is incorrect: Ecological resilience is the capacity of an ecosystem to cope with disturbance
or stress and return to a stable state. This concept measures the amount of stress or disruption
required to transform a system that is maintained by one set of structures and processes to a
different set of structures and functions.
Option b is incorrect: Population viability/thresholds refers to the probability of survival of a
population/species in the face of ecological processes such as disturbance. When the amount of
habitat available declines below the “extinction threshold”, a population/species will decline and
eventually disappear; in addition to habitat for particular populations, a species‟ survival depends on
maintaining healthy genetic variability.
Option d is incorrect: A keystone species is an organism that helps define an entire ecosystem.
Without its keystone species, the ecosystem would be dramatically different or cease to exist
altogether. Keystone species have low functional redundancy
Source: Chapter 1 Forum Red Book Environment
Subject:) Environment
Subtopic:) Ecology and Ecosystems

Q.4) Consider the following:


1. Protozoa.
2. Virus.
3. Bacteria.
4. Lightning and thunder.
O

How many of the above play a crucial role in converting atmospheric nitrogen into a compound
C
S.

effectively utilized by plants?


TE
O
FN

Forum Learning Centre: Delhi - 2nd Floor, IAPL House, 19 Pusa Road, Karol Bagh, New Delhi - 110005 | Patna - 2nd floor, AG Palace, E Boring Canal
Road, Patna, Bihar 800001 | Hyderabad - 1st & 2nd Floor, SM Plaza, RTC X Rd, Indira Park Road, Jawahar Nagar, Hyderabad, Telangana 500020
D

9311740400, 9311740900 | https://academy.forumias.com | admissions@forumias.academy | helpdesk@forumias.academy


.P
W

[2]
W
W
SFG 2024 | LEVEL 1 | Test #41 – Solutions |

a) Only one
b) Only two
c) Only three
d) All four

Ans) b
Exp) Option b is the correct answer.
Nitrogen gas (N2) makes up about 78% of Earth's atmosphere, but most organisms cannot use it
directly. Nitrogen fixation is the process by which atmospheric nitrogen is converted into a form that
can be utilized by living organisms.
Option 1 is incorrect: Protozoa are single-celled organisms that are primarily predators and don't
possess the ability to fix atmospheric nitrogen.
Option 2 is incorrect: Viruses lack the necessary machinery for nitrogen fixation and rely on host
cells for their own replication. Nitrogen fixation is a complex biological process carried out by certain
microorganisms, particularly nitrogen-fixing bacteria. These bacteria possess the enzyme
nitrogenase, which allows them to convert atmospheric nitrogen (N2) into ammonia (NH3) or other
nitrogen compounds that can be used by plants and other organisms.
Viruses are much simpler entities compared to bacteria and lack the cellular machinery necessary for
metabolic processes like nitrogen fixation. Viruses are essentially genetic material (either DNA or
RNA) surrounded by a protein coat. They lack the cellular structures and metabolic pathways found in
bacteria or other living organisms.
Option 3 is correct: Specific bacteria possess the capability to convert atmospheric nitrogen (N2) into
ammonia (NH3). Free-living nitrogen-fixing bacteria like Azotobacter, convert N2 into ammonia
(NH3). Ammonia (NH3) in turn is converted into nitrite (NO2-) and then into nitrate (NO3-) by
nitrifying bacteria such as Nitrosomonas and Nitrobacter.
Option 4 is correct: The intense heat generated during astronomical events like lightning and
thunder can disrupt the bonds of atmospheric nitrogen molecules. The resulting free nitrogen atoms
then combine with oxygen in the air, forming nitrogen oxides. These oxides dissolve in moisture,
creating nitrates that are transported to the Earth's surface through precipitation.
Source: Forum IAS Environment Red book - Chapter 1
Subject:) Environment
Subtopic:) Ecology and Ecosystems

Q.5) Consider the following statements:


1. Tundra region is found in the higher latitudes near the polar regions only.
2. Cotton grass and lichens are the vegetation found in the arctic Tundra region.
3. Mammals in the tundra region have small tails and small ears.
How many of the statements given above are correct?
a) Only one
b) Only two
c) All three
d) None

Ans) b
Exp) Option b is the correct answer.
Tundra ecosystem is the ecosystem that is devoid of trees and covered with snow for most of the
year. They are found in cold climates and in the regions with limited or scarce rainfall.
Statement 1 is incorrect: There are two types of tundra arctic and alpine. Arctic tundra occurs below
O

the polar ice cap. It occupies the northern fringe of Canada, Alaska, European Russia, Siberia and
C
S.

island group of Arctic Ocean. Alpine tundra occurs at high mountains at all latitudes.
TE
O
FN

Forum Learning Centre: Delhi - 2nd Floor, IAPL House, 19 Pusa Road, Karol Bagh, New Delhi - 110005 | Patna - 2nd floor, AG Palace, E Boring Canal
Road, Patna, Bihar 800001 | Hyderabad - 1st & 2nd Floor, SM Plaza, RTC X Rd, Indira Park Road, Jawahar Nagar, Hyderabad, Telangana 500020
D

9311740400, 9311740900 | https://academy.forumias.com | admissions@forumias.academy | helpdesk@forumias.academy


.P
W

[3]
W
W
SFG 2024 | LEVEL 1 | Test #41 – Solutions |

Statement 2 is correct: Typical vegetation of arctic tundra is cotton grass, sedges, dwarf heath,
willows, birches and lichens. Animals of tundra are reindeer, musk ox, arctic hare, caribous, lemmings
and squirrels.
Statement 3 is correct: Organisms in Tundra have evolved to withstand extreme environments.
Mammals of the tundra region have large body size, small tail and small ear to avoid the loss of heat
from the surface.
Source: https://ncert.nic.in/textbook/pdf/iess105.pdf
Shankar IAS Environment Book Chapter 3
Subject:) Environment
Subtopic:) Terrestrial and Aquatic Ecosystem

Q.6) Consider the following statements regarding „Kelp forests‟:


1. They are underwater forests dominated by brown algae.
2. They occur only in tropical warm waters as they cannot thrive in cold waters.
3. They play a crucial role in reducing the rate of Coastal erosion.
4. They are a valuable source of alginates that are used in various food industries.
How many of the statements given above are correct?
a) Only one
b) Only two
c) Only three
d) All four

Ans) c
Exp) Option c is the correct answer.
Kelp are large brown algae found in the oceans. Kelp forests are underwater areas with a high density
of kelp which provide food and shelter for thousands of fish, invertebrates, and marine mammal
species.
Statement 1 is correct: Kelp forests are underwater ecosystems dominated by large brown algae,
resembling terrestrial forests. Because of their dependency upon light for photosynthesis, kelp forests
form in shallow open waters and are rarely found deeper than 49-131 feet.
Statement 2 is incorrect: Kelp are large brown algae that live in cool, relatively shallow waters and
nutrient-rich waters close to the shore. They are found primarily near temperate and polar regions
and occasionally in tropical waters. Temperate and polar regions provide the ideal temperature
range for the growth of most kelp species. Upwelling in these regions provides an abundant supply of
nutrients, including nitrogen and phosphorus, which are essential for kelp growth.
Statement 3 is correct: Kelps grow in dense groupings much like a forest on land. Kelp forests alter
the movement of water and can provide a buffer against storm surges by reducing wave energy. This
helps reduce the removal of sediments from beaches and reduces the rate of coastal erosion.
Statement 4 is correct: Kelp is a valuable source of alginates, used as thickeners and stabilizers in
various food and industrial applications. Algin is a complex carbohydrate derived from brown algae
(Kelp is a brown algae) and it is used as food additives.
Source: Forum IAS Environment Red book - Chapter 1
https://oceanservice.noaa.gov/facts/kelp.html
https://www.wildlifetrusts.org/habitats/marine/kelp-beds-and-forests
Subject:) Environment
Subtopic:) Ecology and Ecosystems
O
C
S.
TE
O
FN

Forum Learning Centre: Delhi - 2nd Floor, IAPL House, 19 Pusa Road, Karol Bagh, New Delhi - 110005 | Patna - 2nd floor, AG Palace, E Boring Canal
Road, Patna, Bihar 800001 | Hyderabad - 1st & 2nd Floor, SM Plaza, RTC X Rd, Indira Park Road, Jawahar Nagar, Hyderabad, Telangana 500020
D

9311740400, 9311740900 | https://academy.forumias.com | admissions@forumias.academy | helpdesk@forumias.academy


.P
W

[4]
W
W
SFG 2024 | LEVEL 1 | Test #41 – Solutions |

Q.7) With reference to biotic interactions of living organisms, consider the following pairs:
Types of Examples
interactions
1. Mutualism Honeybees and
flowers
2. Commensalism A large tree shades
a small plant
3. Amensalism Cow dung and dung
beetles.
4. Parasitism Virus inside human
body
How many of the pairs given above are correctly matched?
a) Only one
b) Only two
c) Only three
d) All four

Ans) b
Exp) Option b is the correct answer.
Biotic interactions refer to the relationships and connections between living organisms within an
ecosystem. These interactions play a fundamental role in shaping the structure and functioning of
biological communities. Biotic interactions can be categorized into various types, including
mutualism, commensalism, parasitism, and competition
Pair 1 is correct: Mutualism is a symbiotic relationship in which both species involved benefit from
interaction. For example, the interaction between Honeybees and flowers benefits both species.
Bees benefit from obtaining nectar from flowers and the pollination activities of bees are essential for
the reproduction of numerous plant species.
Pair 2 is incorrect: Commensalism is a type of relationship between two species in which one species
benefit, and the other is neither harmed nor benefited. A relationship between Cow dung and dung
beetles is an example of commensalism. While cow dung provides food and shelter to dung beetles,
beetles have no effect on the cows.
A large tree shading a small plant is an example of Amensalism, not Commensalism.
Pair 3 is incorrect: Amensalism is a type of interaction in which one organism is negatively affected,
and the other is unaffected. For example, a large tree shading a small plant, negatively affecting the
growth of the small plant and large tree was neither benefited nor harmed by this relation.
Relation between Cow dung and dung beetles displays Commensalism, not Amensalism.
Pair 4 is correct: Parasitism is a relationship in which one organism, the parasite, benefits at the
expense of the host organism, usually by deriving nutrients from it. Viruses inside the human body
are a classic example of parasitism. In this process, Viruses depend on host cells for replication and
may cause harm to the host in the process.
Source: https://ncert.nic.in/ncerts/l/lebo113.pdf
Forum IAS Environment Red book - Chapter 1
Subject:) Environment
Subtopic:) Ecology and Ecosystems

Q.8) Consider the following statements regarding an environmental pollutant:


1. It is water-soluble.
2. It is biologically active.
O

3. It is immobile.
C
S.
TE
O
FN

Forum Learning Centre: Delhi - 2nd Floor, IAPL House, 19 Pusa Road, Karol Bagh, New Delhi - 110005 | Patna - 2nd floor, AG Palace, E Boring Canal
Road, Patna, Bihar 800001 | Hyderabad - 1st & 2nd Floor, SM Plaza, RTC X Rd, Indira Park Road, Jawahar Nagar, Hyderabad, Telangana 500020
D

9311740400, 9311740900 | https://academy.forumias.com | admissions@forumias.academy | helpdesk@forumias.academy


.P
W

[5]
W
W
SFG 2024 | LEVEL 1 | Test #41 – Solutions |

How many of the above characteristics are essential for an environmental pollutant to undergo
biomagnification?
a) Only one
b) Only two
c) All three
d) None

Ans) a
Exp) Option a is the correct answer.
Biomagnification is the phenomenon where pollutants tend to accumulate at higher concentrations as
they progress through successive trophic levels. For example, the increasing concentration of DDT
(dichloro-diphenyl-trichloroethane) in organisms as they move up the trophic levels in a food chain.
Option 1 is incorrect: If the pollutant is soluble in water, it will be easily excreted by the organism
in form of urine or sweat and cannot concentrate to a harmful level in an organism. For
biomagnification to occur, pollutants must be soluble in fats as pollutants that dissolve in fats may be
retained for a long time in an organism.
Option 2 is correct: For biomagnification to occur a pollutant must be biologically active otherwise
it cannot be passed onto the next level of food chains.
Option 3 is incorrect: A pollutant must be mobile for biomagnification to occur. If it is not mobile, it
will stay in one place and is unlikely to be taken up by organisms. If it is not consumed by organisms
in large numbers, it may not reach the concentration level required for it to store in organisms.
Source: https://ncert.nic.in/ncerts/l/lebo116.pdf
Forum IAS Environment Red book - Chapter 1
https://www1.biologie.uni-hamburg.de/b-
online/library/marietta/2bioma95.html#:~:text=long%2D-,lived,-mobile
Subject:) Environment
Subtopic:) Ecology and Ecosystems

Q.9) With reference to the characteristics of humus, consider the following statements:
Statement-I: Humus undergoes decomposition at an extremely slow rate.
Statement-II: Humus is highly resistant to microbial action.
Which one of the following is correct in respect of the above statements?
a) Both Statement-I and Statement-II are correct and Statement-II is not the correct explanation for
Statement-I.
b) Both Statement-I and Statement-II are correct and Statement-II is the correct explanation for
Statement-I.
c) Statement-I is incorrect but Statement-II is correct.
d) Statement-I is correct but Statement-II is incorrect.

Ans) b
Exp) Option b is the correct answer.
The process of humification involves the breakdown of organic residues by microorganisms,
leading to the formation of humus. Humus is characterized by its dark color, high organic carbon
content, and ability to bind with soil particles. It improves soil structure, promotes nutrient
availability to plants, and helps in retaining moisture.
Statement-I is correct: Humus is a dark colored amorphous substance. It is highly resistant to
microbial action and undergoes decomposition at an extremely slow rate.
Statement-II is correct and explains the Statement-I: Humus is highly resistant to microbial action
O

due to its complex structure. Certain substances within humus act as natural defenses against
C
S.

microbial degradation, contributing to its slow decomposition rate. Consequently, humus stands out
TE
O
FN

Forum Learning Centre: Delhi - 2nd Floor, IAPL House, 19 Pusa Road, Karol Bagh, New Delhi - 110005 | Patna - 2nd floor, AG Palace, E Boring Canal
Road, Patna, Bihar 800001 | Hyderabad - 1st & 2nd Floor, SM Plaza, RTC X Rd, Indira Park Road, Jawahar Nagar, Hyderabad, Telangana 500020
D

9311740400, 9311740900 | https://academy.forumias.com | admissions@forumias.academy | helpdesk@forumias.academy


.P
W

[6]
W
W
SFG 2024 | LEVEL 1 | Test #41 – Solutions |

as the most stable organic molecule found in the soil. These attributes explain the delayed
decomposition of humus in comparison to other organic matter.
Source: https://ncert.nic.in/textbook/pdf/lebo114.pdf (Pg no 243 & 244)
Subject:) Environment
Subtopic:) Ecology and Ecosystems

Q.10) Which of the following statements explains the meaning of the „Carrying Capacity‟ of an
ecosystem?
a) It refers to the amount of resources consumed by each individual within the ecosystem.
b) It is the maximum population size that an ecosystem can support without degrading itself.
c) It refers to the total size of renewable resources available in the ecosystem.
d) It is the sum of all the physical and chemical factors that a species needs to survive in an
ecosystem.

Ans) b
Exp) Option b is the correct answer.
In ecological terms, an ecosystem's carrying capacity is the population size that can be supported
indefinitely upon its available resources and services. In other words, it is defined as the maximum
population size that an ecosystem can sustainably support without degrading itself.
Knowledge base:
The limit of an ecosystem depends on three factors:
1) the number of resources available in the ecosystem;
2) the size of the population or community; and
3) the amount of resources each individual within the community is consuming
Source: https://ncert.nic.in/ncerts/l/lebo113.pdf
https://education.nationalgeographic.org/resource/resource-library-carrying-capacity/
Subject:) Environment
Subtopic:) Ecology and Ecosystems

Q.11) Which of the following have species that can establish symbiotic relationship with other
organisms?
1. Cnidarians
2. Fungi
3. Protozoa
Select the correct answer using the code given below:
a) 1 and 2 only
b) 2 and 3 only
c) 1 and 3 only
d) 1, 2 and 3

Ans) d
Exp) Option d is the correct answer.
Some organisms live together and share both shelter and nutrients. This relationship is called
symbiosis.
Statement 1 is correct. Cnidaria is an invertebrate phylum which often involves in a symbiotic
relation with the unicellular dinoflagellate algae, called zooxanthellae.
Statement 2 is correct. Certain fungi live inside the roots of plants. The plants provide nutrients to
the fungus, and, in return, the fungus provides water and certain nutrients. Fungi also lives in a
O

symbiotic relation with algae as a lichen.


C
S.
TE
O
FN

Forum Learning Centre: Delhi - 2nd Floor, IAPL House, 19 Pusa Road, Karol Bagh, New Delhi - 110005 | Patna - 2nd floor, AG Palace, E Boring Canal
Road, Patna, Bihar 800001 | Hyderabad - 1st & 2nd Floor, SM Plaza, RTC X Rd, Indira Park Road, Jawahar Nagar, Hyderabad, Telangana 500020
D

9311740400, 9311740900 | https://academy.forumias.com | admissions@forumias.academy | helpdesk@forumias.academy


.P
W

[7]
W
W
SFG 2024 | LEVEL 1 | Test #41 – Solutions |

Statement 3 is correct. Protozoa also forms symbiotic relations with other organisms. Symbiosis in
protozoa mostly represents:
1) a close mutualistic association between a protozoan and unicellular symbiont (like bacteria,
cyanobacteria or/and unicellular algae) or
2) protozoans and a multicellular organism (ruminants, lower termites, wood-eating cockroaches,
plants)
Source: UPSC CSE PRE. 2021
Subject:) Environment
Subtopic:) Ecology and Ecosystems

Q.12) In the context of Ecology, which one of the following best describes the meaning of the term
'Population'?
Population is a group of-
a) individuals of the same species living in the same geographical area at a given point of time.
b) different species living in the same geographical area at a given point of time.
c) organisms that can interbreed and reproduce with one another at a given point of time.
d) all living organisms that inhabit the earth at a given point of time.

Ans) a
Exp) Option a is the correct answer.

Option a is correct: Population refers to a group of individuals usually of the same species,
occupying a defined area during a specific time.
Option b is incorrect: A Community encompasses all populations of various species coexisting in
the same region and interacting with one another. It constitutes all biotic factors within a site.
Option c is incorrect: Species are defined as a group of organisms sharing similar characteristics,
capable of interbreeding and reproducing.
Option d is incorrect: Biodiversity pertains to the diversity of living species on Earth,
encompassing plants, animals, bacteria, and fungi. It broadly characterizes the extensive variety of life
on Earth. It can be used more specifically to refer to all of the species in one region or ecosystem
Source: Forum IAS Environment Red book - Chapter 1
Subject:) Environment
Subtopic:) Ecology and Ecosystems

Q.13) Consider the following statements regarding „PARIVESH‟ portal of the Government of India:
1. It has been developed by the Ministry of Environment, Forest and Climate Change (MoEF&CC).
O

2. It provides for online submission and monitoring of the proposals seeking Environment and
C

Wildlife clearance.
S.
TE
O
FN

Forum Learning Centre: Delhi - 2nd Floor, IAPL House, 19 Pusa Road, Karol Bagh, New Delhi - 110005 | Patna - 2nd floor, AG Palace, E Boring Canal
Road, Patna, Bihar 800001 | Hyderabad - 1st & 2nd Floor, SM Plaza, RTC X Rd, Indira Park Road, Jawahar Nagar, Hyderabad, Telangana 500020
D

9311740400, 9311740900 | https://academy.forumias.com | admissions@forumias.academy | helpdesk@forumias.academy


.P
W

[8]
W
W
SFG 2024 | LEVEL 1 | Test #41 – Solutions |

Which of the statements given above is/are correct?


a) 1 only
b) 2 only
c) Both 1 and 2
d) Neither 1 nor 2

Ans) c
Exp) Option c is the correct answer.
PARIVESH (Proactive and Responsive facilitation by Interactive, Virtuous and Environmental
Single-window Hub) serves as a Single-Window Integrated Environmental Management System.
This portal is in consonance with the 'Digital India' initiative of the Government of India and a step in
the direction of ensuring the principle of „Minimum Government and Maximum Governance.‟
Statements 1 and 2 are correct: The PARIVESH portal was developed by the Ministry of
Environment, Forest and Climate Change (MoEF&CC) with technical support from the National
Informatics Centre, (NIC). It is a web based, role-based workflow application which has been
developed for online submission and monitoring of the proposals submitted by the proponents for
seeking Environment, Forest, Wildlife and CRZ Clearances from Central, State and district level
authorities. It automates the entire tracking of proposals which includes online submission of a new
proposal, editing/updating the details of proposals and displays status of the proposals at each stage
of the workflow.
Source: https://pib.gov.in/Pressreleaseshare.aspx?PRID=1542607
Subject:) Environment
Subtopic:) Environmental Impact Assessment

Q.14) Consider the following ecosystems:


1. Lake
2. Wetland
3. Savannah Grassland
4. Oceans
5. Desert
Which of the options given below correctly arranges ecosystem in decreasing order of their Net
Primary Productivity (NPP)?
a) 2-1-4-3-5
b) 2-3-4-1-5
c) 2-1-3-5-4
d) 2-3-1-4-5

Ans) d
Exp) Option d is the correct answer.
Net primary productivity (NPP) refers to the rate at which plants and other primary producers
accumulate energy as biomass. This is determined by assessing the growth in biomass per unit area
within a specified time frame.
The comparative order of ecosystems is as follows:
Option 2 - With shallow waters, rich nutrients, and abundant plant life, wetlands boast this group's
highest net primary productivity (NPP).
Option 3- While less productive than wetlands, savannahs still have significant NPP due to efficient
energy capture by grasses. Savannah, unlike Steppe, is categorized as a transitional grassland which
has higher NPP as compared to the non-transitional grasslands.
O

Option 1- Lakes typically have lower NPPs than wetlands and savannahs due to factors like water
C
S.

depth, nutrient levels, and temperature.


TE
O
FN

Forum Learning Centre: Delhi - 2nd Floor, IAPL House, 19 Pusa Road, Karol Bagh, New Delhi - 110005 | Patna - 2nd floor, AG Palace, E Boring Canal
Road, Patna, Bihar 800001 | Hyderabad - 1st & 2nd Floor, SM Plaza, RTC X Rd, Indira Park Road, Jawahar Nagar, Hyderabad, Telangana 500020
D

9311740400, 9311740900 | https://academy.forumias.com | admissions@forumias.academy | helpdesk@forumias.academy


.P
W

[9]
W
W
SFG 2024 | LEVEL 1 | Test #41 – Solutions |

Option 4- The open ocean has a comparatively lower NPP due to limited light penetration and
nutrient availability in deeper waters. However, coastal areas and upwelling zones can be much
more productive.
Option 5- Arid ecosystems with limited rainfall and scarce vegetation, deserts naturally exhibit the
lowest NPP among these ecosystems
Source: Shankar IAS Environment- Chapter 1
Subject:) Environment
Subtopic:) Terrestrial and Aquatic Ecosystem

Q.15) Which of the following statements correctly describes the term „Ecological footprint‟?
a) The amount of carbon-dioxide produced because of any activity by a human or machine.
b) The amount of oxygen required to replenish the environment in a particular area.
c) The total greenhouse gas (GHG) emissions caused by an individual, organization or place
expressed as carbon dioxide equivalent.
d) The area needed to provide the renewable resources that a population consumes and to absorb its
waste.

Ans) d
Exp) Option d is the correct answer.
Ecological Footprint accounting measures the demand on and supply of nature. The ecological
footprint (EF) estimates the biologically productive land and sea area needed to provide the
renewable resources that a population consumes and to absorb the wastes it generates—using
prevailing technology and resource-management practices— rather than trying to determine how
many people a given land area, or the entire planet can support. The amount of carbon-dioxide that
any activity by any human or machine produces is called carbon footprint not ecological footprint.
Source: Shankar IAS Environment Book Chapter 23
Subject:) Environment
Subtopic:) Ecology and Ecosystems

Q.16) With reference to the various Biomes and their characteristics, consider the following pairs:

Biome Characteristics

1. Taiga Highest Productivity


among forest
ecosystems.

2. Tropical Poor nutrient


Rainforests composition of the soil.

3. Savannah Distinct Wet and Dry


Grassland Season.

How many of the above pairs are correctly matched?


a) Only one
b) Only two
c) All three
d) None

Ans) b
O

Exp) Option b is the correct answer.


C
S.
TE
O
FN

Forum Learning Centre: Delhi - 2nd Floor, IAPL House, 19 Pusa Road, Karol Bagh, New Delhi - 110005 | Patna - 2nd floor, AG Palace, E Boring Canal
Road, Patna, Bihar 800001 | Hyderabad - 1st & 2nd Floor, SM Plaza, RTC X Rd, Indira Park Road, Jawahar Nagar, Hyderabad, Telangana 500020
D

9311740400, 9311740900 | https://academy.forumias.com | admissions@forumias.academy | helpdesk@forumias.academy


.P
W

[10]
W
W
SFG 2024 | LEVEL 1 | Test #41 – Solutions |

A biome constitutes the terrestrial segment of the biosphere, distinguished by its climate,
vegetation, animal life, and overall soil composition.

Pair 1 is incorrectly matched- The taiga, found in Northern Europe, Asia, and North America, is
commonly called boreal forests. These forests exhibit lower productivity compared to other forest
ecosystems. Further, the soils in boreal forests are marked by thin podzols, which have a low pH
(acidic) due to extensive leaching.
Pair 2 is correctly matched- Tropical rainforests, located in the equatorial region, are characterized
by a complex structure comprising multiple layers of broad-leafed, tall, closely spaced evergreen
trees. These trees form a continuous canopy overhead. However, the soil in rainforests tends to be
nutrient-poor due to the extensive leaching caused by heavy rains. Despite this, these ecosystems
support a diverse array of species, fostering the coexistence of numerous plant and animal life.
Pair 3 is correctly matched- Savanna regions experience two clearly defined seasons: a wet season
and a dry season. The dry season is characterized by minimal rainfall. In contrast, during the wet
season, there is abundant vegetation growth, featuring lush green grasses and wooded areas.
Moving away from the equator, where rainfall is more pronounced, the grassland becomes
progressively drier, especially during the dry season. The savanna's vegetation consists of scrub,
various grasses, and occasional trees, with the latter typically thriving near water holes, seasonal
rivers, or aquifers. Often referred to as 'Big Game Country,' savannahs draw attention for being
habitats where thousands of animals are hunted for sports and other recreational activities.
Source: Forum IAS, environment red book,Ch1, Pg: 7
Subject:) Environment
Subtopic:) Terrestrial and Aquatic Ecosystem

Q.17) With reference to the Environmental Impact Assessment (EIA), consider the following
statements:
1. The objective of EIA is to foresee the potential environmental problems that would arise out of a
proposed development and address them in the project's planning and design stage.
2. The legal basis for the EIA is provided under the Environmental (Protection) Act, 1986.
Which of the statements given above is/are correct?
a) 1 only
b) 2 only
c) Both 1 and 2
d) Neither 1 nor 2

Ans) c
Exp) Option c is the correct answer.
O
C
S.
TE
O
FN

Forum Learning Centre: Delhi - 2nd Floor, IAPL House, 19 Pusa Road, Karol Bagh, New Delhi - 110005 | Patna - 2nd floor, AG Palace, E Boring Canal
Road, Patna, Bihar 800001 | Hyderabad - 1st & 2nd Floor, SM Plaza, RTC X Rd, Indira Park Road, Jawahar Nagar, Hyderabad, Telangana 500020
D

9311740400, 9311740900 | https://academy.forumias.com | admissions@forumias.academy | helpdesk@forumias.academy


.P
W

[11]
W
W
SFG 2024 | LEVEL 1 | Test #41 – Solutions |

EIA, or Environmental Impact Assessment, in essence, entails a thorough examination of a project's


environmental impact. It functions as a decision-making tool, aiding policymakers in approving,
rejecting, or seeking alternatives for a project.
Statement 1 is correct- EIA incorporates environmental issues into development activities as early
as the feasibility report preparation phase. This allows for the integration of environmental concerns
and mitigation strategies in project development. EIA can often avoid future liability or costly changes
in project design.
EIA aims to identify potential environmental issues and address them during project planning and
design. The EIA/ Environment Management Plan (EMP) helps planners and government agencies
make decisions by identifying significant impacts and mitigating actions.
Statement 2 is correct- Section 3(1)(i) of the Environmental Protection Act, 1986, empowers the
Central Government to take measures to "prevent environmental pollution". Section 3(1)(ii) gives the
power to plan and coordinate environmental monitoring and research. These provisions form the
legal basis for the EIA process established through subsequent notifications.
Source: Forum IAS, environment red book, Ch3, Pg: 27
https://moef.gov.in/wp-content/uploads/2018/04/Introducation.pdf
https://www.thehindu.com/news/national/highways-within-100-km-of-loc-borders-will-not-
need-environmental-clearance-centre/article65657889.ece
https://indianexpress.com/article/india/environment-ministry-amends-eia-rules-exempts-
highways-strategic-importance-8039740/
Subject:) Environment
Subtopic:) Environmental Impact Assessment

Q.18) With reference to the Mangroves Ecosystem, consider the following statements:
Statement-I: Mangrove forests can survive in anaerobic conditions.
Statement-II: Mangroves have a specialized root structure called Pneumatophores.
Which one of the following is correct in respect of the above statements?
a) Both Statement-I and Statement-II are correct and Statement-II is the correct explanation for
Statement-I
b) Both Statement-I and Statement-II are correct and Statement-II is not the correct explanation for
Statement-I
c) Statement-1 is correct but Statement-II is incorrect
d) Statement-l is incorrect but Statement-II is correct

Ans) a
Exp) Option a is the correct answer.
Mangroves are evergreen forests that thrive in sheltered low-lying coasts, estuaries, mudflats, tidal
creek backwaters, marshes, and lagoons within tropical and subtropical regions. These ecosystems
consist of salt-tolerant plants, known as halophytes, and are well-adapted to endure harsh
ecological conditions. Examples of mangrove species include Sonneratia and Avicennia.
Statement I is correct- Mangrove trees have evolved to thrive in anaerobic environments
characterized by waterlogged conditions with minimal to no oxygen. In India, Mangrove forests are
found in the Sundarbans (West Bengal), Pichavaram (Tamil Nadu), Mahanadi, Goa, Krishna
Godavari, Ratnagiri, and the Cauvery Delta.
Statement II is correct, and Statement II is the correct explanation for Statement I- Mangroves can
survive in anaerobic conditions due to the presence of specialized root structures called
Pneumatophores. Pneumatophores are specialized aerial roots that emerge from the underground
roots of mangrove trees and extend upward into the air. They have several important functions that
O

contribute to the adaptation of mangroves to their unique environment, which often includes
C
S.

waterlogged and anaerobic soils in tidal zones.


TE
O
FN

Forum Learning Centre: Delhi - 2nd Floor, IAPL House, 19 Pusa Road, Karol Bagh, New Delhi - 110005 | Patna - 2nd floor, AG Palace, E Boring Canal
Road, Patna, Bihar 800001 | Hyderabad - 1st & 2nd Floor, SM Plaza, RTC X Rd, Indira Park Road, Jawahar Nagar, Hyderabad, Telangana 500020
D

9311740400, 9311740900 | https://academy.forumias.com | admissions@forumias.academy | helpdesk@forumias.academy


.P
W

[12]
W
W
SFG 2024 | LEVEL 1 | Test #41 – Solutions |

Source: Forum IAS, environment red book, Ch1, Pg: 16


Subject:) Environment
Subtopic:) Terrestrial and Aquatic Ecosystem

Q.19) Consider the following kinds of organisms:


1. Bacteria
2. Fungi
3. Flowering plants
Some species of which of the above kinds of organisms are employed as biopesticides?
a) 1 only
b) 2 and 3 only
c) 1 and 3 only
d) 1, 2 and 3

Ans) d
Exp) Option d is the correct answer.
All of the above are employed as biopesticides.
Biopesticides include naturally occurring substances that control pests (biochemical pesticides),
microorganisms that control pests (microbial pesticides), and pesticidal substances produced by
plants containing added genetic material (plant-incorporated protectants) or PIPs.
Bacteria and Fungi are more targeted in their activity than conventional chemicals. For example, a
certain fungus might control certain weeds, and another fungus might control certain insects. The
most common microbial biopesticide is Bacillus thuringiensis.
Flowering Plants can also be employed as biopesticides eg. Neem is the best example of flowering
plant that can be used as a biopesticide.
Source: UPSC CSE PRE. 2012
Subject:) Environment
Subtopic:) Ecology and Ecosystems

Q.20) With reference to Sulphur cycle, consider the following statements:


1. It is a sedimentary cycle as it does not involve gaseous components.
2. Sulphur is taken by plants in the form of sulphates.
Which of the statements given above is/are correct?
a) 1 only
b) 2 only
c) Both 1 and 2
d) Neither 1 nor 2

Ans) b
Exp) Option b is the correct answer.
Statement 1 is incorrect. Although the Sulphur cycle is mostly sedimentary, it involves gaseous
components like Hydrogen Sulphide and Sulphur dioxide. Organisms like marshes and tidal flats
release Sulphur in the form of Hydrogen Sulphide (a gaseous component) by anaerobic respiration.
Sources like volcanic eruption, fossil fuels release large amounts of Sulphur dioxide (a gaseous
component) in the atmosphere.
Statement 2 is correct. Whatever the source, Sulphur is absorbed by the plant in the form of
sulphate. It is then incorporated through a series of metabolic processes into Sulphur bearing amino
acid incorporated in the proteins of autotroph tissues. It then passes through the grazing food chain.
O

Source: Shankar IAS Environment Book Chapter 2


C
S.

Subject:) Environment
TE
O
FN

Forum Learning Centre: Delhi - 2nd Floor, IAPL House, 19 Pusa Road, Karol Bagh, New Delhi - 110005 | Patna - 2nd floor, AG Palace, E Boring Canal
Road, Patna, Bihar 800001 | Hyderabad - 1st & 2nd Floor, SM Plaza, RTC X Rd, Indira Park Road, Jawahar Nagar, Hyderabad, Telangana 500020
D

9311740400, 9311740900 | https://academy.forumias.com | admissions@forumias.academy | helpdesk@forumias.academy


.P
W

[13]
W
W
SFG 2024 | LEVEL 1 | Test #41 – Solutions |

Subtopic:) Ecology and Ecosystems

Q.21) “If rainforests and tropical forests are the lungs of the Earth, then surely wetlands function as its
kidneys.” Which one of the following functions of wetlands best reflects the above statement?
a) The water cycle in wetlands involves surface runoff, subsoil percolation and evaporation.
b) Algae form the nutrient base upon which fish, crustaceans, molluscs, birds, reptiles and mammals
thrive.
c) Wetlands play a vital role in maintaining sedimentation balance and soil stabilization.
d) Aquatic plants absorb heavy metals and excess nutrients.

Ans) d
Exp) Option d is the correct answer.
Just as forests are called as the ‘lungs of the earth’, wetlands are the ‘kidneys’ that regulate water
and filter waste from the landscape.
Wetlands include mangroves, peat lands and marshes, lakes, deltas, floodplains and flooded forests
and even coral reefs. Aquatic plants of wetlands absorb heavy metals and excessive nutrients thereby
purifying water. For instance, Mangrove trees have the ability of storing metals, transferring these
elements from the sediment and concentrating them in their tissues. They can serve as a means for
the immobilization and removal of pollutants.
Source: UPSC CSE. PRE. 2022
Subject:) Environment
Subtopic:) Terrestrial and Aquatic Ecosystem

Q.22) With reference to the various examples of food chain, consider the following statements:
1. At the lowest tropic level, the grazing Food Chain starts with plants whereas the Detritus Food
Chain starts with dead organic matter.
2. Instead of Grazing Food Chain, the Detritus food chain acts as a major conduit of energy flow in an
aquatic ecosystem.
Which of the statements given above is/are correct?
a) 1 only
b) 2 only
c) Both 1 and 2
d) Neither 1 nor 2

Ans) a
Exp) Option a is the correct answer.
A food chain represents the sequential transfer of matter and energy in the form of food from one
organism to another. It consistently begins with producers and concludes with the apex predator.
Two primary types of food chains exist: The Grazing Food Chain and the Detritus Food Chain.
Statement 1 is correct- The Grazing Food Chain starts with living producers (plants). However, the
Detritus food chain begins with the decomposition of dead organic matter by microorganisms,
including bacteria and fungi.
Statement 2 is incorrect- In aquatic ecosystems, the Grazing Food Chain serves as the primary
pathway for energy flow. In contrast, in terrestrial ecosystems, a significantly greater proportion of
energy flows through the detritus food chain compared to the Grazing Food Chain.
O
C
S.
TE
O
FN

Forum Learning Centre: Delhi - 2nd Floor, IAPL House, 19 Pusa Road, Karol Bagh, New Delhi - 110005 | Patna - 2nd floor, AG Palace, E Boring Canal
Road, Patna, Bihar 800001 | Hyderabad - 1st & 2nd Floor, SM Plaza, RTC X Rd, Indira Park Road, Jawahar Nagar, Hyderabad, Telangana 500020
D

9311740400, 9311740900 | https://academy.forumias.com | admissions@forumias.academy | helpdesk@forumias.academy


.P
W

[14]
W
W
SFG 2024 | LEVEL 1 | Test #41 – Solutions |

Source:https://ncert.nic.in/textbook/pdf/lebo114.pdf
Subject:) Environment
Subtopic:) Ecology and Ecosystems

Q.23) Consider the following statements:


1. The pioneer species are those species which initiate the development of an ecological community
in an area with currently no life form‟s existence.
2. Pioneer species can tolerate harsh environmental conditions.
3. In pioneer community, the plants and animals are in balance with each other and their
environment.
How many of the statements given above are incorrect?
a) Only one
b) Only two
c) All three
d) None

Ans) a
Exp) Option a is the correct answer.
Succession is when one community of plants and animals replaces another in an ecosystem. During
succession some species colonise an area and their population become more numerous whereas
populations of other species decline and even disappear. The entire sequence of communities that
successively change in a given area are called sere(s).
Statement 1 is correct. The pioneer species are those species which initiate the development of an
ecological community in an area with currently no life form’s existence. In primary succession on
rocks these are usually lichens which are able to secrete acids to dissolve rock, helping in weathering
and soil formation.
Statement 2 is correct. Pioneer species can tolerate and trade through most of the prevailing harsh
environmental conditions.
Statement 3 is incorrect. In a climax community (not in pioneer community), the plants and animals
are in balance with each other and their environment. A climax community refers to a stable
ecosystem in its final stage of ecological succession. The climax community remains stable as long as
the environment remains unchanged.
Source: https://www.sciencedirect.com/topics/earth-and-planetary-sciences/pioneer-species
https://www.biologyonline.com/dictionary/pioneer-species
NCERT Class 12 Biology – Chapter 14, P 251.
Subject:) Environment
Subtopic:) Ecology and Ecosystem

Q.24) With reference to Sundarbans Forest located in West Bengal, consider the following statements:
1. It is bound to its west by the river Muriganga.
O

2. It is the only mangrove forest in the world inhabited by Tigers.


C
S.

3. Sundarbans National Park is included in the UNESCO‟s World Heritage Site list.
TE
O
FN

Forum Learning Centre: Delhi - 2nd Floor, IAPL House, 19 Pusa Road, Karol Bagh, New Delhi - 110005 | Patna - 2nd floor, AG Palace, E Boring Canal
Road, Patna, Bihar 800001 | Hyderabad - 1st & 2nd Floor, SM Plaza, RTC X Rd, Indira Park Road, Jawahar Nagar, Hyderabad, Telangana 500020
D

9311740400, 9311740900 | https://academy.forumias.com | admissions@forumias.academy | helpdesk@forumias.academy


.P
W

[15]
W
W
SFG 2024 | LEVEL 1 | Test #41 – Solutions |

How many of the above statements are correct?


a) Only one
b) Only two
c) All three
d) None

Ans) c
Exp) Option c is the correct answer.
The Sundarbans, a cluster of low-lying islands in the Bay of Bengal, are renowned for their
distinctive mangrove forests, spanning across India and Bangladesh. Encompassing an area
exceeding 40,000 square kilometers, this dynamic delta region stands as one of the largest in the
world.
Statement 1 is correct- Situated in the coastal region of the Bay of Bengal, Sundarbans is an
extensive contiguous mangrove forest ecosystem that extends across India and Bangladesh within
the delta of the Ganges, Brahmaputra, and Meghna rivers—the largest delta globally. It is bound on
the west by the river Muriganga and on the east by the rivers Harinbhahga and Raimangal. Other
major rivers flowing through this ecosystem are Saptamukhi, Thakuran, Matla and Goasaba.
Statement 2 is correct- The Sundarbans Delta stands as the sole mangrove forest globally where
tigers reside.
Statement 3 is correct- Established in 1984, the Sundarbans National Park forms a central area
within the tiger reserve and earned UNESCO World Heritage status in 1987.
Source: https://en.unesco.org/biosphere/aspac/sunderban
Subject:) Environment
Subtopic:) Terrestrial and Aquatic Ecosystem

Q.25) An ecosystem is the basic unit for the scientific study of nature. The ecosystem is made up of
two inseparable components which are biotope and biocenosis. In this context, consider the following
statements:
1. Biocenosis represents the non-living components of an ecosystem with specific physical
characteristics.
2. Biotope represents the living components of an ecosystem where living organisms are in constant
interaction.
Which of the statements given above is/are correct?
a) 1 only
b) 2 only
c) Both 1 and 2
d) Neither 1 nor 2

Ans) d
Exp) Option d is the correct answer.
Ecosystem is a biological system (biogeocenosis), consisting of a community of living organisms
(biocenosis), their habitat (biotope), interacting as a system, with its own metabolism and energy
exchange.
Statement 1 is incorrect: Biotope (not biocenosis) represents the non-living components of an
ecosystem. The soil, water, air, and all the physio-chemical processes going on in an ecosystem
comes under the term biotope.
Statement 2 is incorrect: Biocenosis (not Biotope) is a set of living organisms such as animals, plants
or microorganisms, that are in constant interaction and are, therefore, in a situation of
O

interdependence.
C
S.
TE
O
FN

Forum Learning Centre: Delhi - 2nd Floor, IAPL House, 19 Pusa Road, Karol Bagh, New Delhi - 110005 | Patna - 2nd floor, AG Palace, E Boring Canal
Road, Patna, Bihar 800001 | Hyderabad - 1st & 2nd Floor, SM Plaza, RTC X Rd, Indira Park Road, Jawahar Nagar, Hyderabad, Telangana 500020
D

9311740400, 9311740900 | https://academy.forumias.com | admissions@forumias.academy | helpdesk@forumias.academy


.P
W

[16]
W
W
SFG 2024 | LEVEL 1 | Test #41 – Solutions |

Source:
https://readindiabooks.com/ebook/PDF/1568993705_Environment___Ecology_By_Majid_Hussai
n_3rd_Edition.pdf
https://www.thedailyeco.com/biotopes-and-biocenosis-definition-and-differences-
63.html#:~:text=Ecology%20is%20the%20study%20of,of%20living%20beings%20called%20biocenosi
s
Subject:) Environment
Subtopic:) Ecology and Ecosystems

Q.26) Consider the following statements regarding the „Forest Advisory Committee‟:
1. It is the final authority to approve the proposals for the diversion of forest land for non-forest uses
in India.
2. It is a statutory body incorporated under the Forest Rights Act of 2006.
3. The Minister of Environment, Forest and Climate Change is the ex-officio Chairperson of this
Committee.
How many of the above statements are correct?
a) Only one
b) Only two
c) All three
d) None

Ans) d
Exp) Option d is the correct answer.
Forest Advisory Committee (FAC) is a specialized body under the administrative jurisdiction of the
Ministry of Environment, Forests, and Climate Change (MoEFCC) to ensure the judicious use of
forested lands.
Statement 1 is incorrect: The primary function of the Forest Advisory Committee is to review and
advise the government on the proposals for the diversion of forest land for non-forest purposes.
When any project or activity requires the use of forest land, such as for mining, infrastructure
development, or industrial projects, a proposal is submitted to the FAC for examination. The
committee assesses the environmental impact, biodiversity concerns, and other relevant factors
before making recommendations or decisions regarding diversion. The recommendations of FAC are
put before the State Government and then the Central Government for final approval.
Statement 2 is incorrect: The Forest Advisory Committee is not incorporated under the Forest
Rights Act of 2006. Instead, it operates under the Forest (Conservation) Act of 1980. The Forest
Rights Act of 2006 is a separate legislation that deals with the recognition and vesting of forest rights
and responsibilities of local communities.
Statement 3 is incorrect: The Director General of Forests, Ministry of Environment and Forests is
the chairperson of the Forest Advisory Committee. Other members in include:
1) The Additional Director General of Forests, Ministry of Environment and Forests
2) The Additional Commissioner (Soil Conservation), Ministry of Agriculture
3) Three non-official members who shall be experts one each in Mining, Civil Engineering and
Development Economics
4) The Inspector General of Forests (Forest Conservation), Ministry of Environment and Forests
Hence, the statement given is incorrect.
Source: Forum IAS, Environment Red Book, Ch1, Pg: 13
https://www.indiacode.nic.in/bitstream/123456789/19381/1/the_forest_%28conservation%29_act
%2C_1980.pdf
O

Subject:) Environment
C
S.

Subtopic:) Environmental Impact Assessment


TE
O
FN

Forum Learning Centre: Delhi - 2nd Floor, IAPL House, 19 Pusa Road, Karol Bagh, New Delhi - 110005 | Patna - 2nd floor, AG Palace, E Boring Canal
Road, Patna, Bihar 800001 | Hyderabad - 1st & 2nd Floor, SM Plaza, RTC X Rd, Indira Park Road, Jawahar Nagar, Hyderabad, Telangana 500020
D

9311740400, 9311740900 | https://academy.forumias.com | admissions@forumias.academy | helpdesk@forumias.academy


.P
W

[17]
W
W
SFG 2024 | LEVEL 1 | Test #41 – Solutions |

Q.27) Consider the following statements:


1. Eurythermal organisms cannot tolerate any change in the temperatures.
2. Hydrophytes are the plants that are able to survive in anaerobic environments.
3. Epiphytes are the plants that use the supporting plants for shelter and not for water or food.
How many of the statements given above are correct?
a) Only One
b) Only two
c) All three
d) None

Ans) b
Exp) Option b is the correct answer.
Adaptation is any attribute of the organism (morphological, physiological, and behavioural) that
enables the organism to survive and reproduce in its habitat. Many adaptations have evolved over a
long evolutionary time and are genetically fixed.
Statement 1 is incorrect: Eurythermal organisms can tolerate and thrive in a wide range of
temperatures. Stenothermal organisms are restricted to a narrow range of temperatures.
Statement 2 is correct: Hydrophytes are plants that adapted to live in water or moist
environments. Adaptations include:
1) In submerged forms, the stem is long, thin, porous, and bendable.
2) Either the cuticle is completely absent or, if present, it is narrow and poorly formed.
3) Hydrophytes are able to survive in anaerobic environments.
4) They have specialized aerating organs.
Statement 3 is correct: Epiphytes are plants that grow by perching on other plants. They solely use
the supporting plants for shelter and not for water or food. These epiphytes can be found in tropical
rain forests. Orchids and Hanging Mosses are examples. Adaptations include:
1) Aerial roots (for respiration) and clinging roots (which hold the epiphytes firmly to the surfaces of
the supporting structures).
2) In some, the stems are succulent (they store water) and produce pseudo-bulbs or tubers.
3) The occurrence of a thick cuticle and depressed stomata slows transpiration significantly.
Source: https://ncert.nic.in/ncerts/l/lebo113.pdf
https://www.bartleby.com/subject/science/biology/concepts/ecological-adaptations
Subject:) Environment
Subtopic:) Ecology and Ecosystems

Q.28) In the context of the Wetlands (Conservation and Management) Rules of 2017, consider the
following:
1. River Channels
2. Paddy fields
3. Man-made water bodies specifically for drinking purposes
4. Structures specifically constructed for aquaculture purposes
5. Marsh ecosystem
How many of the above are not included in the definition of „wetland‟ as per the provisions of the
Wetlands (Conservation and Management) Rules of 2017?
a) Only two
b) Only three
c) Only four
d) All five
O
C
S.

Ans) c
TE
O
FN

Forum Learning Centre: Delhi - 2nd Floor, IAPL House, 19 Pusa Road, Karol Bagh, New Delhi - 110005 | Patna - 2nd floor, AG Palace, E Boring Canal
Road, Patna, Bihar 800001 | Hyderabad - 1st & 2nd Floor, SM Plaza, RTC X Rd, Indira Park Road, Jawahar Nagar, Hyderabad, Telangana 500020
D

9311740400, 9311740900 | https://academy.forumias.com | admissions@forumias.academy | helpdesk@forumias.academy


.P
W

[18]
W
W
SFG 2024 | LEVEL 1 | Test #41 – Solutions |

Exp) Option c is the correct answer.


The Wetlands (Conservation and Management) Rules of 2017 is a legal framework in India aimed at
preserving and managing wetland ecosystems. Enacted under the provisions of the Environment
(Protection) Act, 1986, these rules provide guidelines for the identification, conservation, and
sustainable management of wetlands across the country. The rules define a wetland as an area of
1) Marsh ecosystem (option 5 is incorrect),
2) fen,
3) peatland, or water, whether natural or artificial, permanent or temporary
4) water bodies with static or flowing water, fresh, brackish, or salt, including areas of marine
water with depths not exceeding six meters at low tide.
The rules explicitly exclude certain areas from the definition of wetlands, such as
1) river channels (Option 1 is correct)
2) paddy fields, (Option 2 is correct)
3) man-made water bodies constructed for drinking purposes, and (option 3 is correct)
4) structures specifically constructed for aquaculture, salt production, recreation, and irrigation
purposes. (Option 4 is correct)
Source: https://indianwetlands.in/uploads/Wetlands2017.pdf
Forum IAS, environment red book.
Subject:) Environment
Subtopic:) Terrestrial and Aquatic Ecosystem

Q.29) What would happen if phytoplankton of an ocean is completely destroyed for some reason?
1. The ocean as a carbon sink would be adversely affected.
2. The food chains in the ocean would be adversely affected.
3. The density of ocean water would drastically decrease.
Select the correct answer using the codes given below:
a) 1 and 2 only
b) 2 only
c) 3 only
d) 1, 2 and 3

Ans) a
Exp) Option a is the correct answer.
Statement 1 is correct. Phytoplankton are responsible for most of the transfer of carbon dioxide
from the atmosphere to the ocean. Carbon dioxide is consumed during photosynthesis, and the
carbon is incorporated in the phytoplankton, just as carbon is stored in the wood and leaves of a tree.
Thus, the complete destruction of the phytoplankton of an ocean would release this sequestered
carbon dioxide to the atmosphere and adversely affect the ocean as a carbon sink.
Statement 2 is correct. Phytoplankton and algae form the basis of aquatic food webs. They are eaten
by primary consumers like zooplankton, small fish, and crustaceans. Primary consumers are in turn
eaten by fish, small sharks, corals, and baleen whales. Thus, the complete destruction of the
phytoplankton of an ocean would adversely affect the food chains in the ocean.
Statement 3 is incorrect. Phytoplankton are photosynthesizing microscopic biotic organisms that
inhabit the upper sunlit layer of almost all oceans and bodies of fresh water on Earth. The density of
the ocean water depends on heat content and salinity. Thus, the density of ocean water would not
be much affected if the phytoplankton of the ocean is completely destroyed.
Source: UPSC CSE PRE. 2012
Subject:) Environment
O

Subtopic:) Terrestrial and Aquatic Ecosystem


C
S.
TE
O
FN

Forum Learning Centre: Delhi - 2nd Floor, IAPL House, 19 Pusa Road, Karol Bagh, New Delhi - 110005 | Patna - 2nd floor, AG Palace, E Boring Canal
Road, Patna, Bihar 800001 | Hyderabad - 1st & 2nd Floor, SM Plaza, RTC X Rd, Indira Park Road, Jawahar Nagar, Hyderabad, Telangana 500020
D

9311740400, 9311740900 | https://academy.forumias.com | admissions@forumias.academy | helpdesk@forumias.academy


.P
W

[19]
W
W
SFG 2024 | LEVEL 1 | Test #41 – Solutions |

Q.30) Consider the following statements about Euphotic Zone/ Photic zone:
1. It is the upper layer of the aquatic ecosystems, up to which light penetrates and within which
photosynthetic activity is confined.
2. It is also known as the hadopelagic zone.
3. Common species that can be primarily found in euphotic zones are scallops, sea urchins, shrimps
and krills.
How many of the statements given above are correct?
a) Only one
b) Only two
c) All three
d) None

Ans) a
Exp) Option a is the correct answer.
The euphotic/photic zone is the uppermost or “well-lit” layer of the ocean in which there is sufficient
light energy available to support net photosynthetic growth.
Statement 1 is correct: The euphotic/photic zone is the upper layer of the aquatic ecosystems, up to
which light penetrates and within which photosynthetic activity is confined. Both photosynthesis and
respiration activity take place in this zone.
Statement 2 is incorrect: Extending from ocean‟s surface down to about 200 meters, the
euphotic/photic zone supports photosynthesis and hence known as epipelagic zone. The hadopelagic
zone is found in deep ocean trenches below 6000 m.
Statement 3 is incorrect: The euphotic/photic zone is home to phytoplankton, zooplankton and
nekton. Species like diatoms, dinoflagellates, cyanobacteria, coccolithophore, cryptomonads and
silicoflagellates, along with zooplankton like copepods and other crustaceans can be found in this
zone. Whereas, scallops, sea urchins, shrimps, krill, etc. can be primarily found in the aphotic zone.
Source:
https://ncert.nic.in/pdf/publication/sciencelaboratorymanuals/classXII/biology/lelm212.pdf
Shankar IAS Environment Book Chapter 4
Subject:) Environment
Subtopic:) Terrestrial and Aquatic Ecosystem

Q.31) Which of the following statements best describes “carbon fertilization”?


a) Increased plant growth due to increased concentration of carbon dioxide in the atmosphere.
b) Increased temperature of Earth due to increased concentration of carbon dioxide in the
atmosphere.
c) Increased acidity of oceans as a result of increased concentration of carbon dioxide in the
atmosphere.
d) Adaptation of all living beings on Earth to the climate change brought about by the increased
concentration of carbon dioxide in the atmosphere.

Ans) a
Exp) Option a is the correct answer.
Carbon fertilization is the increased growth of plants due to increased rate of photosynthesis in
plants that results from increased levels of carbon dioxide in the atmosphere.
The effect varies depending on the plant species, the temperature, and the availability of water and
nutrients.
It is also the artificial enrichment of the atmosphere of greenhouses with carbon dioxide used to
O

improve production levels.


C
S.
TE
O
FN

Forum Learning Centre: Delhi - 2nd Floor, IAPL House, 19 Pusa Road, Karol Bagh, New Delhi - 110005 | Patna - 2nd floor, AG Palace, E Boring Canal
Road, Patna, Bihar 800001 | Hyderabad - 1st & 2nd Floor, SM Plaza, RTC X Rd, Indira Park Road, Jawahar Nagar, Hyderabad, Telangana 500020
D

9311740400, 9311740900 | https://academy.forumias.com | admissions@forumias.academy | helpdesk@forumias.academy


.P
W

[20]
W
W
SFG 2024 | LEVEL 1 | Test #41 – Solutions |

Elevated CO2 concentration increases the plant growth and leaf photosynthesis, even under water
stress conditions. Also, elevated CO2 concentration enhances water use efficiency under water
stresses.
Source: UPSC CSE PRE. 2018
Subject:) Environment
Subtopic:) Terrestrial and Aquatic Ecosystem

Q.32) In every summer we see a rise in the events of Forest Fires. In this context consider the
following:
1. Recycling of Nutrients.
2. Proliferation of Invasive species.
3. Reduction in the competition among the existing tree species.
4. Facilitating seed germination.
5. Creating diverse habitats for plants and animals.
How many of the above given are the consequences of forest fire?
a) Only two
b) Only three
c) Only four
d) All five

Ans) c
Exp) Option c is the correct answer.
Forest fires are a complex and fascinating phenomenon with both destructive and regenerative
capabilities.
Option 1 is correct: As dead or decaying plants begin to build up on the ground, they may prevent
organisms within the soil from accessing nutrients or block animals on the land from accessing the
soil. This coating of dead organic matter can also choke outgrowth of smaller or new plants. Forest
fire helps the nutrients released from the burned material return more quickly into the soil. In this
way, fire helps recycle nutrients.
Option 2 is incorrect: The animal casualties from wildfires are low as animals survive by burrowing
into the ground or fleeing to safer areas. Conversely, fires can help rid an ecosystem of invasive
species that have not adapted to regular wildland fires.
Option 3 is correct: Forest fire reduces competition among the trees, allowing existing trees to grow
larger. It control the encroachment of undesirable plants and encourage desirable food plants such as
legumes for both forage and soil improvements, or shrubs.
Option 4 is correct: Several plants require fire to move along their life cycles. For example, seeds
from many pine tree species are enclosed in pine cones that are covered in pitch, which must be
melted by fire for the seeds to be released. Other trees, plants, and flowers, like certain types of lilies,
also require fire for seed germination.
Option 5 is correct: Different types of fire can create diverse habitats for various plants and animals.
Some fires burn hot and fast, clearing out dense undergrowth, while others burn slowly and cool,
creating openings in the forest canopy. This variety of habitats benefits a wider range of species.
Source: Forum IAS, environment red book,Ch1, Pg: 11
https://education.nationalgeographic.org/resource/ecological-benefits-fire/
https://www.cseindia.org/understanding-eia-383#:~:text=EIA%20and%20SEA).-
,Table%203%3A%20Difference%20in%20EIA%20and%20SEA,-
Environment%20impact%20assessment
Subject:) Environment
O

Subtopic:) Terrestrial and Aquatic Ecosystem


C
S.
TE
O
FN

Forum Learning Centre: Delhi - 2nd Floor, IAPL House, 19 Pusa Road, Karol Bagh, New Delhi - 110005 | Patna - 2nd floor, AG Palace, E Boring Canal
Road, Patna, Bihar 800001 | Hyderabad - 1st & 2nd Floor, SM Plaza, RTC X Rd, Indira Park Road, Jawahar Nagar, Hyderabad, Telangana 500020
D

9311740400, 9311740900 | https://academy.forumias.com | admissions@forumias.academy | helpdesk@forumias.academy


.P
W

[21]
W
W
SFG 2024 | LEVEL 1 | Test #41 – Solutions |

Q.33) Consider the following statements with reference to Harmful Algal Blooms (HAB):
1. It can be caused by the upwelling of seawater.
2. It can lead to discoloration of water.
3. It can lead to an increase of dinoflagellates and diatoms in water bodies.
How many of the statements given above are correct?
a) Only one
b) Only two
c) All three
d) None

Ans) c
Exp) Option c is the correct answer.
Harmful algal blooms, or HABs, occur when colonies of algae grow out of control and produce toxic
or harmful effects on people, fish, shellfish, marine mammals and birds.
Statement 1 is correct. Upwelling brings deep, nutrient-rich water to the surface. These nutrients,
like nitrogen and phosphorus, are essential for algal growth. The cold, nutrient-rich water combined
with sunlight and warm surface temperatures can create ideal conditions for certain types of algae to
rapidly multiply, forming blooms. Thus, upwelling of seawater is one of the factors which triggers HAB
Statement 2 is correct. HABs can discolor and degrade water quality, making it unsuitable for
swimming, drinking, and recreational activities. This can affect tourism and local economies.
Statement 3 is correct. Cyanobacteria, a type of photosynthetic bacteria also known as blue-green
algae, are often the cause of algal blooms in freshwater and occasionally in marine water.
Alternatively, dinoflagellates and diatoms, different types of phytoplankton, are the most common
HAB species in marine and brackish waters, including estuaries.
Source: Forum IAS, environment red book, Ch 1, Pg: 16
Subject:) Environment
Subtopic:) Terrestrial and Aquatic Ecosystem

Q.34) Which of the following statements best describes the term „Standing Crop‟ in the context of
Ecology?
a) It is the biomass of all primary producers in a unit area at a specific moment in time.
b) It is the total biomass of crops sown in a hectare at a specific moment in time.
c) It is the biomass of all trees in a forest ecosystem at a specific moment in time.
d) It is the biomass of all living organisms in a unit area at a specific moment in time.

Ans) d
Exp) Option d is the correct answer.
The standing crop of an ecosystem includes all living organisms, regardless of their position in the
food chain. Standing crops can be measured for different levels of organization within an ecosystem,
from individual organisms to populations, communities, and entire ecosystems.
In ecology, standing crop refers to the total amount of living biomass present in a given area at a
specific moment in time. This includes all organisms, from producers like plants and algae to
consumers like animals and bacteria. Standing crop is an important metric for understanding the
productivity and functioning of an ecosystem. It can be used to assess the carrying capacity of an
environment, track changes over time, and compare different ecosystems. The biomass of a species is
expressed in terms of fresh or dry weight.
Source: Pg: 247, https://ncert.nic.in/textbook/pdf/lebo114.pdf
Subject:) Environment
O

Subtopic:) Ecology and Ecosystems


C
S.
TE
O
FN

Forum Learning Centre: Delhi - 2nd Floor, IAPL House, 19 Pusa Road, Karol Bagh, New Delhi - 110005 | Patna - 2nd floor, AG Palace, E Boring Canal
Road, Patna, Bihar 800001 | Hyderabad - 1st & 2nd Floor, SM Plaza, RTC X Rd, Indira Park Road, Jawahar Nagar, Hyderabad, Telangana 500020
D

9311740400, 9311740900 | https://academy.forumias.com | admissions@forumias.academy | helpdesk@forumias.academy


.P
W

[22]
W
W
SFG 2024 | LEVEL 1 | Test #41 – Solutions |

Q.35) Consider the following statements regarding the „shola‟ forests:


1. These are categorized as montane wet temperate forests.
2. These forests can be found in the Nilgiris ranges of Tamil Nadu and Kerala.
Which of the statements given above is/are correct?
a) 1 only
b) 2 only
c) Both 1 and 2
d) Neither 1 nor 2

Ans) c
Exp) Option c is the correct answer.
Statement 1 is correct: The shola forests are characterized as ‘Southern Montane Wet Temperate
Forest’ by various experts like Harry George Champion and SK Seth. These forests are found in valleys
with sufficient moisture and proper drainage, generally at an altitude of more than 1500 meters. The
top regions of these forests are covered with grasslands, known as Shola grasslands.
Statement 2 is correct: Shola forests are distributed across various parts of the Western Ghats. Shola
forests are found in the Nilgiris, located in the states of Tamil Nadu and Kerala. These forests are
also found in the Anamalais, Palni hills, Kalakadu, Mundanthurai and Kanyakumari in the states of
Tamil Nadu and Kerala.
Source: https://ncert.nic.in/ncerts/l/kegy105.pdf
https://www.downtoearth.org.in/blog/forests/why-south-india-needs-the-shola-forests-of-the-
nilgiris-68948
Subject:) Environment
Subtopic:) Terrestrial and Aquatic Ecosystem

Q.36) Read the following paragraph carefully and answer the question that follows:
“When Arizona‟s Grand Canyon was formed, it acted as a huge physical barrier and resulted in the
separation of the species of squirrels into two distinct groups of almost equal sizes. The barrier made
it impossible for the species to breed with one another. Now, each species, equally dominant,
develops differently based on the demand of their unique habitat.”
Which of the following type of speciation most appropriately describes the above paragraph?
a) Allopatric Speciation
b) Peripatric Speciation
c) Parapatric Speciation
d) Artificial Speciation

Ans) a
Exp) Option a is the correct answer.
Speciation is how a new kind of plant or animal species is created. Speciation occurs when a group
within a species separates from other members of its species and develops its own unique
characteristics.
Allopatric speciation occurs when a species separates into two separate groups due to which they
get isolated from one another. A physical barrier, such as a mountain range or a waterway, makes it
impossible for them to breed with one another. So, the given case, formation of Grand Canyon
created a physical barrier for squirrels and other small mammals to interbreed. Today, two separate
squirrel species inhabit the north and south rims of the canyon.
Source:
https://www.nationalgeographic.org/encyclopedia/speciation/#:~:text=When%20Arizona's%20Gra
O

nd%20Canyon%20formed,squirrel%20population%20underwent%20allopatric%20speciation
C
S.

Subject:) Environment
TE
O
FN

Forum Learning Centre: Delhi - 2nd Floor, IAPL House, 19 Pusa Road, Karol Bagh, New Delhi - 110005 | Patna - 2nd floor, AG Palace, E Boring Canal
Road, Patna, Bihar 800001 | Hyderabad - 1st & 2nd Floor, SM Plaza, RTC X Rd, Indira Park Road, Jawahar Nagar, Hyderabad, Telangana 500020
D

9311740400, 9311740900 | https://academy.forumias.com | admissions@forumias.academy | helpdesk@forumias.academy


.P
W

[23]
W
W
SFG 2024 | LEVEL 1 | Test #41 – Solutions |

Subtopic:) Ecology and Ecosystems

Q.37) Consider the following statements about ecological pyramids:


1. It does not include the possibility of a species to be present at two or more trophic levels.
2. It does not accommodate a food web.
3. Saprophytes are often not included in ecological pyramids.
How many of the above are limitations of ecological pyramids?
a) Only one
b) Only two
c) All three
d) None

Ans) c
Exp) Option c is the correct answer.
Ecological pyramids show the relative amounts of various parameters (such as number of organisms,
energy, and biomass) across trophic levels.

Ecological Pyramid

Food Web
Statement 1 is correct. Many species occupy multiple trophic levels in a food web, depending on
their life stage, diet, or other factors. Ecological pyramids typically depict a simplified food chain and
may not accurately represent this complexity. Therefore, this acts as a limitation of ecological
pyramids.
Statement 2 is correct. Ecosystems function through intricate food webs with multiple
interlinkages between species at different levels. Ecological pyramids, focused on a single food chain,
fail to capture this web-like structure and the dynamic predator-prey relationships within it.
Statement 3 is correct. Saprophytes, also known as decomposers, play a crucial role in breaking
down dead organic matter and returning nutrients to the ecosystem. However, they are often not
included in ecological pyramids, despite their significant contribution to energy flow and nutrient
O

cycling.
C
S.

Source: https://ncert.nic.in/textbook/pdf/lebo114.pdf
TE
O
FN

Forum Learning Centre: Delhi - 2nd Floor, IAPL House, 19 Pusa Road, Karol Bagh, New Delhi - 110005 | Patna - 2nd floor, AG Palace, E Boring Canal
Road, Patna, Bihar 800001 | Hyderabad - 1st & 2nd Floor, SM Plaza, RTC X Rd, Indira Park Road, Jawahar Nagar, Hyderabad, Telangana 500020
D

9311740400, 9311740900 | https://academy.forumias.com | admissions@forumias.academy | helpdesk@forumias.academy


.P
W

[24]
W
W
SFG 2024 | LEVEL 1 | Test #41 – Solutions |

Subject:) Environment
Subtopic:) Ecology and Ecosystems

Q.38) In the context of Banni Grasslands, consider the following statements:


1. It is found in Kachchh district in the state of Gujarat.
2. It represents the combination of wetland and grassland ecosystem at the same place.
3. Bhutia tribe dominate the Banni grassland area.
4. It has been approved to host a Cheetah conservation breeding center.
How many of the statements given above are correct?
a) Only two
b) Only three
c) All four
d) None

Ans) b
Exp) Option b is the correct answer.
Statement 1 is correct: Banni grasslands lie in the Kutch district of Gujarat, India. It forms a
transitional zone between the Thar Desert to the north and the Arabian Sea to the south, creating a
mosaic of arid and semi-arid ecosystems. The Banni grassland in Gujarat‟s Kachchh district is one of
the largest grasslands in the Indian subcontinent with an area of over 2500 sq.km
Statement 2 is correct: Banni grasslands cover 2,497 square kilometers and account for over 45% of
Gujarat's pastures. Bannis is classified as a Dichahnthium-Cenchrus-Lasiurus grass cover because it
combines two ecosystems: wetlands and grasslands. These grasslands support a diverse range of
flora and wildlife, including plants, birds, animals, reptiles, and amphibians. A huge freshwater lake
locally known as Chhari-Dhand (Dhand means a shallow lake) is a prominent feature of the Banni
grassland.
Statement 3 is incorrect: Maldharis tribe breed cattle, camels and smaller ruminants like goats and
sheep. They are concentrated mainly in the eastern Kutch as well as in western Kutch which includes
Banni, Asia’s largest grassland. Maldharis dominates the banni grassland.
Statement 4 is correct: Central government has approved a proposal to set up a cheetah
conservation breeding center in Kutch district. Banni grassland used to be the habitat of cheetahs,
which became extinct over time.
Source: Forum IAS, environment red book, Ch 1, Pg: 16
https://bannigrassland.org/banni/
https://indianexpress.com/article/cities/rajkot/there-is-water-and-fodder-now-dont-migrate-
pm-modis-appeal-to-maldharis-7871088/
https://gec.gujarat.gov.in/Home/Bannigrassland
https://indianexpress.com/article/cities/ahmedabad/cheetahs-gujarat-centre-breeding-centre-
banni-grassland-9060967/
Subject:) Environment
Subtopic:) Terrestrial and Aquatic Ecosystem

Q.39) Which one of the following terms describes NOT only the physical space occupied by an
organism, but also its functional role in the community of organisms?
a) Ecotone
b) Ecological niche
c) Habitat
d) Home range
O
C
S.

Ans) b
TE
O
FN

Forum Learning Centre: Delhi - 2nd Floor, IAPL House, 19 Pusa Road, Karol Bagh, New Delhi - 110005 | Patna - 2nd floor, AG Palace, E Boring Canal
Road, Patna, Bihar 800001 | Hyderabad - 1st & 2nd Floor, SM Plaza, RTC X Rd, Indira Park Road, Jawahar Nagar, Hyderabad, Telangana 500020
D

9311740400, 9311740900 | https://academy.forumias.com | admissions@forumias.academy | helpdesk@forumias.academy


.P
W

[25]
W
W
SFG 2024 | LEVEL 1 | Test #41 – Solutions |

Exp) Option b is the correct answer.


Ecological niche is a term for the position of a species within an ecosystem, describing both the range
of conditions necessary for persistence of the species, and its ecological role in the ecosystem.
Source: UPSC CSE PRE. 2013
Subject:) Environment
Subtopic:) Ecology and Ecosystems

Q.40) Consider the following statements regarding aquatic ecosystems:


1. The Lentic aquatic system includes freshwater streams, springs and rivers.
2. Estuaries are more productive as compared to the adjacent sea or river.
Which of the statements given above is/are correct?
a) 1 only
b) 2 only
c) Both 1 and 2
d) Neither 1 nor 2

Ans) b
Exp) Option b is the correct answer.
Statement 1 is incorrect. Lentic aquatic systems are still or stagnant water bodies. They include
pools, ponds, some swamps, bogs and lakes. Whereas lotic water systems are moving water bodies.
They include freshwater streams, springs, rivulets, creeks, brooks, and rivers.
Statement 2 is correct. In estuaries, fresh water from rivers meets ocean water and the two are
mixed by action of tides. Estuaries are highly productive as compared to the adjacent river or sea.
Coastal bays, river mouths and tidal marshes form the estuaries.
Source: Shankar IAS Environment Book Chapter 1
Subject:) Environment
Subtopic:) Terrestrial and Aquatic Ecosystem

Q.41) Due to some reasons, if there is a huge fall in the population of species of butterflies, what could
be its likely consequence/consequences?
1. Pollination of some plants could be adversely affected.
2. There could be a drastic increase in the fungal infections of some cultivated plants.
3. It could lead to a fall in the population of some species of wasps, spiders and birds.
Select the correct using the code given below:
a) 1 only
b) 2 and 3 only
c) 1 and 3 only
d) 1, 2 and 3

Ans) c
Exp) Option c is the correct answer.
Statement 1 is correct: Butterflies play a crucial role in pollination.
Statement 2 is incorrect: There is no drastic increase in the fungal infections of some cultivated
plants.
Statement 3 is correct: Some species of wasps, spiders and birds are natural predators of butterflies.
A fall in population of butterflies could hamper their population.
Source: UPSC CSE PRE. 2017
Subject:) Environment
O

Subtopic:) Ecology and Ecosystems


C
S.
TE
O
FN

Forum Learning Centre: Delhi - 2nd Floor, IAPL House, 19 Pusa Road, Karol Bagh, New Delhi - 110005 | Patna - 2nd floor, AG Palace, E Boring Canal
Road, Patna, Bihar 800001 | Hyderabad - 1st & 2nd Floor, SM Plaza, RTC X Rd, Indira Park Road, Jawahar Nagar, Hyderabad, Telangana 500020
D

9311740400, 9311740900 | https://academy.forumias.com | admissions@forumias.academy | helpdesk@forumias.academy


.P
W

[26]
W
W
SFG 2024 | LEVEL 1 | Test #41 – Solutions |

Q.42) With reference to the Phosphorous cycle, consider the following statements:
1. It is a sedimentary biogeochemical cycle.
2. Unlike the Carbon cycle, there is no respiratory release of Phosphorous into the atmosphere.
3. Phosphorus is added to the environment by weathering rocks.
How many of the above statements are correct?
a) Only one
b) Only two
c) All three
d) None

Ans) c
Exp) Option c is the correct answer.
A bio-geo-chemical cycle denotes the route through which a chemical substance travels across the
biotic and abiotic components of the Earth. It is divided into three parts: Nutrient Cycle, Gaseous
Cycle, and Sedimentary Cycle. The Phosphorus cycle and Sulphur cycle fall under the classification
of sedimentary cycles.
Statement 1 is correct- The phosphorus cycle is a sedimentary biogeochemical process that
illustrates the transfer of phosphorus across the lithosphere, hydrosphere, and biosphere. The cycle
encompasses the flow of phosphorus through rocks, water, soil, sediments, and organisms.
Statement 2 is correct- Phosphorus primarily circulates in its solid forms (phosphates) and doesn't
readily enter the atmosphere through respiration like carbon dioxide does.
Statement 3 is correct: Phosphorus is naturally stored in rock as phosphates. Weathering rocks
release small amounts of phosphates into the soil, which plants take through roots. Herbivores and
other animals acquire phosphorus from plants. Phosphate-solubilizing bacteria decompose waste
products and deceased organisms, releasing phosphorus into the environment.

Source: Pg: 254


https://ncert.nic.in/textbook/pdf/lebo114.pdf
Subject:) Environment
Subtopic:) Ecology and Ecosystems

Q.43) In the context of India State of Forest Report (ISFR), 2021, which of the following statements is
incorrect?
a) Arunachal Pradesh has recorded the maximum amount of forest carbon stock.
b) Assam has recorded the maximum area under bamboo cultivation.
c) West Bengal has the maximum area covered under the mangrove vegetation.
d) Mizoram has the maximum forest area categorized as an „extreme fire-prone‟ zone.

Ans) b
Exp) Option b is the correct answer.
O

The India State of Forest Report, 2021 (ISFR-2021) provides information on forest cover, tree cover,
C

mangrove cover. Moreover, it provides details about carbon stock in India‟s forests, forest fire
S.
TE
O
FN

Forum Learning Centre: Delhi - 2nd Floor, IAPL House, 19 Pusa Road, Karol Bagh, New Delhi - 110005 | Patna - 2nd floor, AG Palace, E Boring Canal
Road, Patna, Bihar 800001 | Hyderabad - 1st & 2nd Floor, SM Plaza, RTC X Rd, Indira Park Road, Jawahar Nagar, Hyderabad, Telangana 500020
D

9311740400, 9311740900 | https://academy.forumias.com | admissions@forumias.academy | helpdesk@forumias.academy


.P
W

[27]
W
W
SFG 2024 | LEVEL 1 | Test #41 – Solutions |

monitoring, forest cover in tiger reserve areas along with above ground estimates of biomass and
climate change hotspots in Indian forests.
Statement a is correct: As per ISFR 2021, forest carbon stock is comprised of soil organic carbon,
Above Ground Biomass (AGB), Below Ground Biomass (BGB), litter, and dead wood. Arunachal
Pradesh has maximum forest carbon stock of 1,023.84 million Tonnes followed by Madhya Pradesh
(609.25 million Tonnes), and Chhattisgarh (496.44 million Tonnes). Jammu & Kashmir has the highest
per hectare of carbon stock. Overall, there has been an increase in the forest carbon stock as
compared to the previous report of 2019.
Statement b is incorrect: As per ISFR, 2021, the total bamboo bearing area in India has decreased by
1.06 million ha. At present, Madhya Pradesh has the largest bamboo bearing area (1.84 million ha),
followed by Arunachal Pradesh (1.57 million ha), and Maharashtra (1.35 million ha). Also, Madhya
Pradesh (2,473 sq. km.) has recorded the highest decrease in the bamboo bearing area.
Statement c is correct: Total mangrove cover in India is 4,992 sq km, which is 0.15% of country’s
geographical area. An increase of 17 sq Km in mangrove cover has been observed as compared to the
previous assessment of ISFR. West Bengal has the highest area under the mangrove vegetation,
followed by Gujarat.
Statement d is correct: Nearly 10.66% area of forest cover in India is under extremely to very
highly fire prone zone. States under the northeastern region show the highest tendency of forest
fire. Mizoram has the highest area under the extreme forest fire prone zone. Hence, the given
statement is correct.
Source: https://fsi.nic.in/isfr-2021/chapter-9.pdf
https://fsi.nic.in/isfr-2021/chapter-8.pdf
https://fsi.nic.in/isfr-2021/chapter-5.pdf
Subject:) Environment
Subtopic:) Terrestrial and Aquatic Ecosystem

Q.44) Consider the following functions:


1. Purification of water.
2. Providing habitat for marine organisms.
3. To provide suitable location for development of seaports.
4. To act as natural buffers for the coastal areas against storms.
How many of the above are the functions of the Estuarine Ecosystem?
a) Only one
b) Only two
c) Only three
d) All four

Ans) d
Exp) Option d is the correct answer.
An estuary is a location where a river or stream meets the sea, representing one of the most
productive water bodies of the world.
Statement 1 is correct- Estuaries function as natural filters, purifying water as it flows through
areas like swamps and salt marshes. During this process, sediments and pollutants are effectively
removed, resulting in cleaner and clearer water.
Statement 2 is correct: Estuaries with their sheltered waters, abundant food sources, and a mix of
freshwater and saltwater provide the perfect nursery grounds for various marine organisms. Fish,
shellfish, crustaceans, and even some reptiles use estuaries to spawn, lay eggs, and raise their young
before venturing out into the open ocean.
O
C
S.
TE
O
FN

Forum Learning Centre: Delhi - 2nd Floor, IAPL House, 19 Pusa Road, Karol Bagh, New Delhi - 110005 | Patna - 2nd floor, AG Palace, E Boring Canal
Road, Patna, Bihar 800001 | Hyderabad - 1st & 2nd Floor, SM Plaza, RTC X Rd, Indira Park Road, Jawahar Nagar, Hyderabad, Telangana 500020
D

9311740400, 9311740900 | https://academy.forumias.com | admissions@forumias.academy | helpdesk@forumias.academy


.P
W

[28]
W
W
SFG 2024 | LEVEL 1 | Test #41 – Solutions |

Statement 3 is correct- Estuaries aid port development through sheltered waters, providing calm
conditions for ship maneuvering, freshwater access for various industrial needs, navigable channels
deepened through dredging, and available flat land for constructing port facilities.
Statement 4 is correct- Estuaries serve as vital natural buffers in multiple ways. During flooding
events, they function as expansive sponges, absorbing excess water and mitigating potential
damage to local habitats and communities. Additionally, estuaries and their adjacent wetlands play a
crucial role in stabilizing shorelines. Furthermore, the presence of wetland plants and soils enables
the absorption of floodwaters and the dissipation of storm surges, contributing to enhanced coastal
resilience.
Source: Forum IAS, environment red book, Ch1, Pg: 16
Subject:) Environment
Subtopic:) Terrestrial and Aquatic Ecosystem

Q.45) Consider the following factors:


1. Excessive use of fertilizers and pesticides
2. Excessive Mining of natural resources
3. Rampant Urbanization
4. Over-drafting of Groundwater
How many of the above given factors can promote the process of desertification of land in the long
run?
a) Only one
b) Only two
c) Only three
d) All four

Ans) d
Exp) Option d is the correct answer.
Desertification is not the natural expansion of existing deserts but the degradation of land in arid,
semi-arid, and dry sub-humid areas. It is a gradual process of soil productivity loss and the thinning
out of the vegetative cover because of human activities and climatic variations.
Option 1 is correct: Excessive use of fertilizers and pesticides to increase crop yields in the near term
frequently results in substantial soil degradation. This could eventually turn fertile land into desert
land.
Option 2 is correct: Mining also leads to land desertification. When a region is excessively mined, the
soil gets damaged significantly, turning it into arid land. Hence, leading to land desertification.
Option 3 is correct: Rampant urbanization due to overpopulation leads to excessive deforestation.
The land without plants becomes prone to various types of erosions. This ultimately leads to
desertification of land.
Option 4 is correct: Over drafting of groundwater is the process in which more groundwater is
extracted from the aquifers than it can be recharged. Depleting groundwater can lead to fall in the
natural water table of the region. This will adversely affect the groundwater-dependent ecosystem,
leading to loss of vegetation cover and land desertification.
Source: https://ncert.nic.in/ncerts/l/hess402.pdf
https://ncert.nic.in/textbook/pdf/hesc105.pdf
Shankar IAS Environment Book Chapter 3
Subject:) Environment
Subtopic:) Terrestrial and Aquatic Ecosystem
O
C
S.
TE
O
FN

Forum Learning Centre: Delhi - 2nd Floor, IAPL House, 19 Pusa Road, Karol Bagh, New Delhi - 110005 | Patna - 2nd floor, AG Palace, E Boring Canal
Road, Patna, Bihar 800001 | Hyderabad - 1st & 2nd Floor, SM Plaza, RTC X Rd, Indira Park Road, Jawahar Nagar, Hyderabad, Telangana 500020
D

9311740400, 9311740900 | https://academy.forumias.com | admissions@forumias.academy | helpdesk@forumias.academy


.P
W

[29]
W
W
SFG 2024 | LEVEL 1 | Test #41 – Solutions |

Q.46) With reference to food chain, which of the following statements correctly explains the
Lindemann law?
a) In a food chain only 10 per cent of the energy is transferred from the Sun to the lowest trophic
level.
b) In the detritus food chain, only 10 per cent of the energy is transferred to each trophic level from
the lower trophic level.
c) In grazing food chain, only 10 per cent of the energy is transferred to each trophic level from the
lower trophic level.
d) In a food chain, minimum 10 per cent of the energy is transferred back to the environment.

Ans) c
Exp) Option c is the correct answer.
The ten percent law of transfer of energy from one trophic level to the next can be attributed to
Raymond Lindeman. But Lindeman did not call it a "law" and instead called it ecological efficiencies.
The number of trophic levels in the grazing food chain is restricted as the transfer of energy follows
10 per cent law – only 10 per cent of the energy is transferred to each trophic level from the lower
trophic level in a grazing food chain.

Knowledge Base:
In detritus food chain the energy flow remains as a continuous passage rather than as a stepwise flow
between discrete entities. Thus, the 10% law is not applicable for the detritus food chain.
The organisms in the detritus food chain are many and include algae, fungi, bacteria, slime moulds,
actinomycetes, protozoa, etc. Detritus organisms ingest pieces of partially decomposed organic
matter, digest them partially and after extracting some of the chemical energy in the food to run their
metabolism, excrete the remainder in the form of simpler organic molecules.
Source: NCERT Class 12 Biology – Chapter 14 – P 247.
Subject:) Environment
Subtopic:) Ecology and Ecosystems

Q.47) Consider the following sites in India:


1. Gulf of Mannar
2. Sunderbans
3. Gulf of Kutch
At how many of the above-mentioned sites, both mangroves and coral reefs can be simultaneously
found?
a) Only one
b) Only two
O

c) All three
C
S.

d) None
TE
O
FN

Forum Learning Centre: Delhi - 2nd Floor, IAPL House, 19 Pusa Road, Karol Bagh, New Delhi - 110005 | Patna - 2nd floor, AG Palace, E Boring Canal
Road, Patna, Bihar 800001 | Hyderabad - 1st & 2nd Floor, SM Plaza, RTC X Rd, Indira Park Road, Jawahar Nagar, Hyderabad, Telangana 500020
D

9311740400, 9311740900 | https://academy.forumias.com | admissions@forumias.academy | helpdesk@forumias.academy


.P
W

[30]
W
W
SFG 2024 | LEVEL 1 | Test #41 – Solutions |

Ans) b
Exp) Option b is the correct answer.
Mangrove ecosystem shows characteristics like salt-tolerance, tidal activity, strong wind velocity,
high temperature and muddy anaerobic soil. They are also known as „Tidal Forests’, ‘Coastal
Woodlands’, ‘Walking Forest in the Sea’, ‘Root of the Sea’ and ‘Oceanic Rain Forests’.
Coral reefs are the skeletons of stony coral polyps cemented together. Coral reefs form the most
dynamic ecosystem, providing shelter and nourishment to marine flora and fauna.
Option 1 is correct: The Gulf of Mannar is a marine biodiversity hotspot with three coastal
ecosystems: coral reefs, seagrass beds, and mangroves. Thus, it is considered as one of the world‟s
richest regions from a marine biodiversity perspective. It is located off the coast of Tamil Nadu,
running down south from Rameswaram to Kanyakumari
Option 2 is incorrect: Sunderbans is one of the largest mangrove ecosystems of the world, located
across India (West Bengal) and Bangladesh. The Sundarbans are at the mouth of the Ganges-
Brahmaputra River system. This region experiences heavy influx of freshwater, which prevents coral
reef formation. For corals to colonize, low siltation, low freshwater inflows by rivers, high salinity,
and optimal temperature are essential. Hence, the given option is incorrect.
Option 3 is correct: Although, the Gulf of Kutch, on the western coast of India, has relatively lesser
coral species than the other reef ecosystems of the world. But, about 40 species of hard coral and 10
species of soft coral live in the protected area. The reserve also contains seven species of mangroves,
important breeding grounds for birds like the painted stork, darter and black-necked Ibis.
Source: https://en.unesco.org/biosphere/aspac/gulf-
mannar#:~:text=The%20Gulf%20of%20Mannar%20endowed,marine%20diversity%20of%20global%2
0significance.
https://www.nccr.gov.in/sites/default/files/GulfofKachch.PDF
Subject:) Environment
Subtopic:) Terrestrial and Aquatic Ecosystem

Q.48) Consider the following statements:


1. Eco-sensitivity is characterized by low levels of resilience that cannot be restored to its original
form easily.
2. An organism's ability to adjust its phenotype to new environmental conditions over its lifetime is
called Acclimatization.
Which of the above statements given above is/are correct?
a) 1 only
b) 2 only
c) Both 1 and 2
d) Neither 1 nor 2

Ans) c
Exp) Option c is the correct answer.
Statement 1 is correct: Eco-sensitivity is expected to have low levels of resilience, and hence is
difficult to be recovered or restored, if disturbed by external influences. The ultimate purpose of
identifying ecological sensitivity or eco-sensitivity is to promote environment-friendly management
regimes and conservation of the ecological wealth of the sites.
Statement 2 is correct: Acclimatization is the compensatory adjustment of organism to change in the
environment, in nature or natural condition. Adjustment to seasonal changes in temperature comes
under acclimatization. It is the ability of an organism to adjust its phenotype to new environmental
conditions over the course of its lifetime.
O

Scientists distinguish between acclimation and acclimatization because the latter adjustment is made
C
S.

under natural conditions when the organism is subject to the full range of changing environmental
TE
O
FN

Forum Learning Centre: Delhi - 2nd Floor, IAPL House, 19 Pusa Road, Karol Bagh, New Delhi - 110005 | Patna - 2nd floor, AG Palace, E Boring Canal
Road, Patna, Bihar 800001 | Hyderabad - 1st & 2nd Floor, SM Plaza, RTC X Rd, Indira Park Road, Jawahar Nagar, Hyderabad, Telangana 500020
D

9311740400, 9311740900 | https://academy.forumias.com | admissions@forumias.academy | helpdesk@forumias.academy


.P
W

[31]
W
W
SFG 2024 | LEVEL 1 | Test #41 – Solutions |

factors. Acclimation, however, refers to a change in only one environmental factor under laboratory
conditions.
Source: https://egyankosh.ac.in/bitstream/123456789/71419/1/Unit-1.pdf
Subject:) Environment
Subtopic:) Ecology and Ecosystems

Q.49) Consider the following statements with reference to Eutrophic lakes and Oligotrophic lakes:
1. Unlike Eutrophic lakes, Oligotrophic lakes are rich in nutrients.
2. Biological Oxygen Demand in Oligotrophic Lake is higher as compared to Eutrophic Lake.
3. Eutrophic lakes typically have reduced water quality whereas oligotrophic lakes have clearer
water.
How many of the statements given above are correct?
a) Only one
b) Only two
c) All three
d) None

Ans) a
Exp) Option a is the correct answer.
Statements 1 is incorrect. Eutrophic lakes are rich in nutrients like nitrogen and phosphorus, often
due to factors like agricultural runoff or sewage discharge. Whereas, Oligotrophic lakes are naturally
low in nutrients, limiting plant and algae growth.
Statements 2 is incorrect. Biological Oxygen Demand is the amount of dissolved oxygen needed (i.e.,
demanded) by aerobic biological organisms to break down organic material present in body. Since
eutrophic lakes have higher levels of biological productivity compared to oligotrophic lakes, eutrophic
lakes have a higher Biological Oxygen Demand due to higher biomass present in them.
Statements 3 is correct. Eutrophic lakes often exhibit reduced water clarity due to the presence of
algae and other suspended particles. Algal blooms can cause water discoloration. Oligotrophic lakes
typically have clearer water, allowing sunlight to penetrate deeper into the water column. This is
because of lower algal concentrations.
Source: https://www.des.nh.gov/sites/g/files/ehbemt341/files/documents/2020-01/bb-3.pdf
https://www.des.nh.gov/sites/g/files/ehbemt341/files/documents/2020-01/laketrophic-explain-
current.pdf
https://ncert.nic.in/ncerts/l/lebo116.pdf
Subject:) Environment
Subtopic:) Terrestrial and Aquatic Ecosystem

Q.50) Within biological communities, some species are important in determining the ability of a large
number of other species to persist in the community. Such species are called:
a) Keystone species
b) Allopatric species
c) Sympatric species
d) Threatened species

Ans) a
Exp) Option a is the correct answer.
In a biological Community, Keystone species is a species whose addition to or loss from an ecosystem
leads to major changes in abundance or occurrence of at least one or more other species. Certain
O

species in an ecosystem is considered more important in determining the presence of many other
C
S.

species in that community/ecosystem. Without keystone species, the ecosystem would be


TE
O
FN

Forum Learning Centre: Delhi - 2nd Floor, IAPL House, 19 Pusa Road, Karol Bagh, New Delhi - 110005 | Patna - 2nd floor, AG Palace, E Boring Canal
Road, Patna, Bihar 800001 | Hyderabad - 1st & 2nd Floor, SM Plaza, RTC X Rd, Indira Park Road, Jawahar Nagar, Hyderabad, Telangana 500020
D

9311740400, 9311740900 | https://academy.forumias.com | admissions@forumias.academy | helpdesk@forumias.academy


.P
W

[32]
W
W
SFG 2024 | LEVEL 1 | Test #41 – Solutions |

dramatically different or cease to exist altogether. Keystone species often include apex predators, but
they can also be other organisms that influence ecological processes profoundly. The concept was
introduced by Robert T. Paine in 1969
Source: Shankar IAS Environment Book Chapter 29 page 345
Subject:) Environment
Subtopic:) Ecology and Ecosystems

O
C
S.
TE
O
FN

Forum Learning Centre: Delhi - 2nd Floor, IAPL House, 19 Pusa Road, Karol Bagh, New Delhi - 110005 | Patna - 2nd floor, AG Palace, E Boring Canal
Road, Patna, Bihar 800001 | Hyderabad - 1st & 2nd Floor, SM Plaza, RTC X Rd, Indira Park Road, Jawahar Nagar, Hyderabad, Telangana 500020
D

9311740400, 9311740900 | https://academy.forumias.com | admissions@forumias.academy | helpdesk@forumias.academy


.P
W

[33]
W
W

You might also like